Sei sulla pagina 1di 126

NAVAL ARCHITECTURE 1

Class Notes

w tonnes
G
G1 d
d G1
G
W tonnes

d
G1

G
w tonnes

Omar bin Yaakob


Naval Architecture Notes

Chapter 1
Introduction
Introduction
To carry out various activities at sea, rivers and lakes, man uses various types of
marine structures, fixed and floating. The structures must be designed and built in
various sizes, shapes and sophistication. Some of them are small and simple such
as a canoe or a raft while others are large and complicated such as an aircraft
carrier or a semi-submersible oil drilling platform.

Naval architecture is an engineering field covering the technology in design of ships


and floating structures. The persons having this expertise are called naval
architects. To build these structures, shipbuilders requires design plans and
guidelines prepared by naval architects. Knowledge in naval architecture is used to
carry out design calculation and to produce plans which can be used by the
shipyards.

Although man has been using marine transport for a long time, not all these
vehicles are designed and constructed using naval architecture knowledge. In fact
the discipline of knowledge on ship design and naval architecture only appeared in
the seventeenth century. Prior to that, shipbuilding is not based on science and
technology but rather on the skills of the master craftsmen.

This dependence on master craftsmen for shipbuilding can be traced back to the
earliest civilization of Egypt, Greek and China. Similarly the war ships and
exploration vessels built by the Romans, Muslims as well as the European colonial
powers were not built using scientific methods.

By the seventeenth century a number of scientists and engineers tried to apply


science and mathematical methods in ship design. Among the earliest was sir
Anthony Deane who wrote Doctrine of Naval Architecture in 1670. Among others, he
put forward a method to determine the draught of the ship before it was built.
Since then, a number of scientists and engineers continued to study and document
various fields of naval architecture. In 1860, a professional body comprising of
naval architects was formed under the name Institution of Naval Architects. A
hundred years later the name was changed to Royal Institution of Naval Architects.

A naval architects works to determine the size and shape of a ship tailored to its
intended use. In addition, he estimates its stability, propulsive power as well as
calculates the size and strength of its structure and the impact of waves on the
vessel. The types of machinery and equipment to be installed, materials to be used
and layout of ship are also determined based on naval architectural knowledge.

Ship hydrostatics and stability is one of the most important subject in Naval
Architecture. The safety of ships, crew, passengers and cargo will be jeopardised if
ships are not stable. In this book, readers will be able to appreciate the basic
terminologies, carry out simple hydrostatics calculations and will be equipped with
basic tools to assess stability of vessels.

© Omar bin Yaakob, July 2008 2


Naval Architecture Notes

Chapter 2
Ship Types, Basic Terms,
Terminologies and Symbols

1. Types of Ships

Ship types can be classed according to:

1. No of Hull

a) Monohull/Single hull
b) Multi-hull
 Catamaran
 Trimaran
 Quadramaran
 Pentamaran

2. Shape of hull form


a) Roundbilge
b) Chine
 Single Chine
 Multiple Chine

3. How the body is supported in water


a) Hydrostatic
b) Hydrodynamic
c) Aeropowered Lift

4. Its function/mission
a) Transport
 Tanker
 Bulk Carrier
 Containership
 Passenger ship
 General Cargo
 LNG Carrier
b) Navy
 Aircrft Carrier
 Submarine
 Frigate
 Destroyer
 Patrol Craft
 Minesweeper

© Omar bin Yaakob, July 2008 3


Naval Architecture Notes

c) Work/Service Vessels
 Tugs
 Supply boat
 Crew Boats
 Heavy Lift
 Crane ships
 Fuel Supply Ships
 Fishing Boat
 Fire Fighting Boats
 Rescue Boats

d) Leisure Vessels
 Cruise ships
 Tourist Boats
 Water Taxi
 Boat Houses

© Omar bin Yaakob, July 2008 4


Naval Architecture Notes

2. Basic Terms, Terminologies and Symbols

After This is represented by a line which is perpendicular to the intersection of


perpendicular(AP) the after edge of the rudder-post with the designed load water-line. This
is the case for both single- and twin-screw merchant ships. For some
classes of warships, and for merchant ships having no rudder-post, the
after perpendicular is taken as the centre-line of the rudder stock.
Amidships ( ) This is the point midway between the forward and after perpendiculars.
Breadth (B) This is the maximum beam, or breadth, of the ship measured at
amidships.
Bilge This is the rounded plating at the lower corners between the vertical shell
plating and the outer bottom plating.
BML Longitudinal metacentric radius measured from centre of buoyancy
BMT Transverse metacentric radius measured from centre of buoyancy
Block coefficient This is a measure of the fullness of the form of the ship and is the ratio of
(CB) the volume of displacement to a given water-line, and the volume of the
circumscribing solid of constant rectangular cross-section having the
same length, breadth and draught as the ship.
ie: CB = ÷ (L x B x T)
The LPP is normally used in calculating the value of CB which varies with
the type of ship.

Fast ships
0.50-0.65 (fine form)
Ordinary ships
0.65-0.75 (moderate form)
Slow ships
0.75-0.85 (full form)
Camber or round of This is the transverse curvature given to the decks, and is measured by
beam the difference between the heights of the deck at side and centre. The
amount of camber amidships is often one-fiftieth of the beam of the ship.
Coefficients of Form is used as a general term to describe the shape of the ship's hull;
form and when comparing one ship's form with another, the naval architect
makes use of a number of coefficients. These coefficients are of great use
in power, stability, strength and design calculations. Examples are Cb,
Cp, Cw etc.
Centre of flotation This is the centre of the area, or centroid, of the water-plane of a ship.
(F) For small angles of trim consecutive water-lines pass through F. The
location is normally on the centerline and longitudinally the distance
from AP or amidships is referred to as LCF
Centre of buoyancy This is the centroid of the underwater form of a ship, and is the point
(B) through which the total force of buoyancy may be assumed to act. Its
position is defined by:
(a) KB the vertical distance above the base, sometimes referred to as VCB
(b) LCB the longitudinal distance measured either from amidships or AP
or FP.
Centre of gravity This is the point through which the total weight of the ship may be
(G) assumed to act. It also is defined by:
(a) KG the vertical distance above the base
(b) LCG the longitudinal distance measured either from amidships or AP
or FP

© Omar bin Yaakob, July 2008 5


Naval Architecture Notes

CP Prismatic coefficient, CP =
AM x L

Depth (D) This is the vertical distance between the base line and the top of the
uppermost continuous deck measured at the side amidships.
Draught (T) This is the depth of immersion from the keel to any waterline.
Displacement This equals the volume ( ) or weight ( ) of water displaced by the hull.
Displacement as a This is volume of water displaced by the ship. It can be imagined as the
volume ( ) volume of the hole in the water occupied by the ship measured in cubic
metres.
Displacement as a This is the weight of water displaced by the ship. It equals the volume
weight ( ) displaced multiplied by a constant representing the density of water, ie:
In fresh water = x 1000 kg/m³
In sea water = x 1025 kg/m³
Weight (or mass) displacement equals the total weight of the ship when
the ship is at rest in equilibrium in still water.
Deadweight This is the difference between the weight displacement and the lightship
weight. This is the measure of a ship's capacity to carry cargo, fuel,
passengers, stores, etc, expressed in tonnes. The size of tankers is often
given in terms of deadweight tonnage, which is the design deadweight.
Ships are usually chartered on the deadweight tonnage.
Displacement This represents the designed total weight of the ship. It is the sum of
tonnage lightship weight and deadweight. The size of warshipsand government
ships is always given in terms of displacement tonnage.
Entrance and run These are the shaped underwater portions of the ship forward and aft of
the parallel middle body.
Forward This is represented by a line which is perpendicular to the intersection of
perpendicular (FP) the designed load water-line with the forward side of the stem.
This may be considered to be the height amidships, of the freeboard deck
Freeboard
at side above the normal summer load water-line.
GML Longitudinal metacentric height measured from centre of gravity
GMT Transverse metacentric height measured from centre of gravity
Gross tonnage This is a measure of the total volume of enclosed spaces in a ship
(GRT) including the under-deck, 'tween-deck spaces and enclosed spaces
above the upper deck. The size of most ordinary merchant ships is
quoted in terms of gross tonnage. Although it unit is tons, it must be
remembered that it is a measure of volume, not weight. 1 ton = 100 ft3.
Heel () This is the amount of inclination of the ship in the transverse direction,
and is usually measured in degrees.
IL Longitudinal moment of inertia of waterplane about amidship
ILCF Longitudinal moment of inertia of waterplane about F
IT Transverse moment of inertia of waterplane about centreline
KML Height of longitudinal metacentre above keel line
KMT Height of transverse metacentre above keel line
Length between This is the horizontal distance between the forward and after
perpendiculars (LPP) perpendiculars.
Length on the This is the length, as measured on the water-line of the ship when
designed load floating in still water in the loaded, or designed, condition.
water-line (LWL)
Length overall (LOA) This is the length measured from the extreme point forward to the

© Omar bin Yaakob, July 2008 6


Naval Architecture Notes

extreme point aft.


Lightship weight This equals the weight of an empty ship i.e. a ship without load. It is
fully equipped and ready to proceed to sea, but with no crew, passengers,
stores, fuel, water, or cargo on board. The boiler or boilers, however, are
filled with water to their working level.
MCT1CM Moment to change trim 1 cm, MCT1CM = GML  BML = pIL
100 L 100L 100L
Midship section This is the transverse section of the ship amidships.
Base line This represents the lowest extremity of the ship. At the point where this
line cuts the midship section a horizontal line is drawn, and it is this line
which acts as the datum, or base line, for all hydrostatic calculations.
Normally, this is the underside of keel.
Midship section This is the ratio of the immersed area of the midship section to the area
area coefficient(CM) of the circumscribing rectangle having a breadth equal to the breadth of
the ship and a depth equal to the draught.
ie: CM = AM ÷ (B x T)
CM values range from about 0.85 for fast ships to 0.99 for slow ships.
Net or register This represents the tonnage of a ship after certain approved deductions,
tonnage ie nonfreight earning spaces, have been made from gross tonnage. A
register ton represents 100 cubic feet of volume.
Parallel middle This is the length over which the midship section remains unchanged.
body (LP)
Prismatic This is the ratio of the volume of displacement of the ship to the volume
coefficient (CP) of the circumscribing solid having a constant section equal to the
immersed midship section area AM, and a length equal to the LPP
i.e. CP = ÷ (AM x L)
The Cp is a measure of the longitudinal distribution of displacement of
the ship, and its value ranges from about 0.55 for fine ships to 0.85 for
full ships.
Rise of floor This is the amount by which the line of the outer bottom plating
amidships rises above the base line, when continued to the moulded
breadth lines at each side.
Sheer This is the curvature given to the decks in the longitudinal direction, and
is measured at any point by the difference between the height at side at
that point and the height at side amidships.
This is the difference between the draughts forward and aft. If the
Trim draught forward is greater than the draught aft it is called trim by the
head, or bow. If the draught aft is greater, it is called trim by the stern.
This is the amount by which the midship section falls in from the half-
Tumble-home
breadth line at any particular depth.
Tonnes per This is the mass which must be added to, or deducted from, a ship in
centimetre (TPC) order to change its mean draught by 1 cm.
Water-plane area This is the ratio of the area of the water-plane to the area of the
coefficient(CWP) circumscribing rectangle having a length equal to the LPP and a breadth
equal to B.
ie: CWP = AW ÷ (L x B)
The range of values is from about 0.70 for a fine ship to 0.90 for a full
ship.

© Omar bin Yaakob, July 2008 7


Naval Architecture Notes

Courtesy http://www.dynagen.co.za/eugene/hulls/terms.html

Exercise:

Visit these websites and get acquainted with more ship terms:
1. http://www.midwestconnection.com/glshpng/glossary.htm
2. http://www.scribd.com/doc/18008262/Ship-Terms-Glossary
3. http://cruises.about.com/od/cruiseglossary/Cruise_Ship_and_Na
utical_Term_Glossary.htm
4. http://www.islandregister.com/terms.html
5. http://phrontistery.info/nautical.html

© Omar bin Yaakob, July 2008 8


Naval Architecture Notes

Chapter 3
Hydrostatics and Floatation
3.1 Archimedes Law of Floatation
Archimedes (born 287 B.C) Law states that
“An object immersed in a liquid experience a lift equivalent to the mass of
liquid the object displaces.”

A man immersed in water for example will feel a weight reduction because part of
the weight is supported by buoyancy. This buoyancy is equal to the weight of water
displaced by his immersed body.

3.2 Reduction of Weight of Immersed Objects


The maximum buoyancy is when the object is fully immersed and this equal the
total outside volume of the object multiplied by the density of the fluid. When
maximum available buoyancy is less than the weight of the object, the object will
sink. That is why an anchor will sink to the bottom. However the object will still feel
the weight reduction.

Example 3.1:

Consider a cuboid having dimensions 1m x 1m x 2m. If it weighs 3 tonnes in air,


what is its apparent weight in water density 1000 kg/m3?

If the object is immersed in liquid, it will


displace liquid around it equivalent to its
external volume. ? tonne

In this case, displaced volume = 1 x 1 x 2 =


2 m3
This is the volume of liquid pushed aside by
the cuboid.

Archimedes says that the weight of this


object in liquid is reduced due to the support
given by liquid on the object. The apparent 2m3
weight equals the weight in air minus the
reduction in weight of the object; or the
buoyancy i.e.

Buoyancy = volume diplacement x density of liquid


= mass displacement
= 2m3 x 1000 kg/m3
= 2000 kg
= 2 tonnes
= reduction in weight

© Omar bin Yaakob, July 2008 9


Naval Architecture Notes
Apparent weight = weight in air – buoyancy

Since the object weighs 3 tonne in air, it will apparently weigh only 1 tonne in
water.

Exercise 3.1

Do similar calculations to find out the apparent weight in oil (density 0.85
tonne/m3) and muddy water (density 1.3 tonne/m3) and mercury (density 13,000
kg/m3)

Fluid Density Fluid Support Apparent Weight


( ) ( ) ( )
Oil
Fresh Water
Muddy Water
Mercury

What can be concluded about relationships between buoyancy of objects


and the densities of fluids in which they are immersed?

3.3 What make a Ship Floats?


When the maximum available buoyancy is more than the weight of the object, the
object will rise to the surface. It will rise to the surface until the weight of the object
balances the buoyancy provided by its immersed portions. When the object is
floating, its buoyancy is just enough to support its weight. At that point:

Total weight W = Buoyancy = Displaced volume x  liquid

This principle explains why a steel or concrete ship can float. As long as the outer
shell of the ship can provide enough volume to displace the surrounding water
exceeding the actual weight of the ship, the ship will float. A floating ship is such
that the total weight of its hull, machinery and deadweight equals to the weight of
water displaced by its outer shell. If, while it is floating weights are added until the
total weight exceeds the maximum buoyancy provided by the outer shell of the ship,
the ship will sink.

3.4 Effect of Density


An object experiences buoyancy force equivalent to the weight of fluid it displaces.
For a particular object, the buoyancy force will depend on the density of the fluid,
since its volume is constant. This explains for example why a bather will feel more
buoyant while swimming at sea compared to in the river or lake. Also, a floating
© Omar bin Yaakob, July 2008 10
Naval Architecture Notes
object of constant weight will sink at a deeper draught in freshwater compared to in
seawater.

Total weight W = Buoyancy = Displaced volume x  liquid

Since weight does not change, the buoyancy is also constant. So displaced volume
will be inversely proportional to the density of fluid. For floating object, this will
determine its level of sinkage or draught.

3.5 Some Simple Problems


The fact that a floating object displaces fluid equivalent to its weight can be used to
solve a number of problems.

Total weight W = Buoyancy = Displaced volume x  water

From this equation, we can obtain the weight of the object if we know the volume of
water displaced. On the other hand, if we know its weight, we can work out its
displaced volume.

Just to understand the concept, consider a floating box of dimension L x B x D,


floating at a draught T.

CASE 1: We know its weight, we can find its draught

In this case, we know the weight of the object, we can find the displaced volume:

Displaced volume = W
water

i.e. for a box-shaped vessel:

Displaced volume = L x B x T
Hence draught T of the cuboid can be found.

© Omar bin Yaakob, July 2008 11


Naval Architecture Notes
Example 3.2

A cuboid shaped wooden block (L x B x D) 1.45m x 0.5m x 0.25m floats in water. If


the block weighs 0.154 tonnes, find its draught if it floats in freshwater density 1.00
tonne/m3.

Solution:

The weight of the block of 0.154 tonnes must be supported by displaced water i.e.
the block must displace 0.154 tonnes of water:

In fresh water,
Volume of displaced water =LxBxT
Weight of displaced water  =  x  FW
 = 1.45 x 0.5 x T x FW

This must equal 0.154 tonne


1.45 x 0.5 x T x fw = 0.154 tonnes
T = 0.212 m

Exercise 3.2

Do similar calculations for salt water (density 1025 kg/m3 and oil density 0.85 tonne/m3)

CASE 2: If we know its draught, we can know its volume displacement, we can
find its weight

If we know the draught of the cuboid, we can find its volume displacement and
hence the weight of the object;

Say if we know its draught T, volume displacement = L x B x T


Weight = Buoyancy = Volume Displacement x  water
Weight = L x B x T x  water

Example 3.3

A box barge length 100m breadth 20m floats at a draught of 5m in sea water 1.025
tonne/m3. Find its weight.

Solution

While floating in sea water density 1.025 tonne/m3:


Volume Displacement = =LxBxT
Weight of barge = Weight displacement, 
W =  =  x salt water
= 100 x 20 x 5 x 1.025
= 10250 tonnes
© Omar bin Yaakob, July 2008 12
Naval Architecture Notes

Exercise 3.3

A block of wood length 5m, breadth 0.5m and depth 0.2m is floating in seawater
at a draught of 0.1m. Find the weight of the block.

Exercise 3.4

Find the new draught of the box in example 3.3 when it goes into river, water
density 1.000 tonne/m3. Also find a new draught if it is in sea water with density
1.100 tonne/m3.

Exercise 3.5 1.0m

A cylindrical container weighing 5 tonne floats


with its axis vertical. If the diameter is 1.0m, find
its draught in:
i. sea water
ii. oil of density 870 kg/ m3.

Exercise 3.6

A cylindrical tank diameter 0.6m and mass 200kg floats with its axis vertical.
Find its present draught in oil ( = 0.95 tonne/m3).
Find the weight of cargo to be added to ensure it will float at a draught of
0.85m.

3.6 Hydrostatic Particulars

A floating object will be at a certain draught depending on the total weight of the
object, density of water and the shape of the object. For a ship, the shape of the
object has strong influence on the draught of the ship; the shape and draught have
to provide enough buoyancy to support the ship.

© Omar bin Yaakob, July 2008 13


Naval Architecture Notes

When a ship is floating at a certain draught, we can find the mass displacement
and weight of the ship if we can find its displaced volume . Also we can know its
waterplane area, calculate its TPC, KB, Cb etc. These particulars which are
properties of the immersed part of the ship are called hydrostatic particulars.
Examples of hydrostatic particulars are:

, , KB, LCB, Aw, BMT, BML, TPC, CB, CP, CM, CW, LCF, MCTC, WSA

These particulars describe the characteristics of the underwater portion of ship at a


particular draught. It is related to volumes, areas, centroids of volumes and areas
and moments of volumes and areas of the immersed portion. If the ship is out of
water, and draught becomes zero, the particulars ceased to exist.

As long as draught and trim is maintained, the size and shape of the underwater
immersed parts of the ship remains the same. The volumes, areas and moments of
areas and volumes remain the same. Once draught or trim changes, the particulars
will also change.

This change in draught will normally occur due to changes in total weight of the
ship, or if a force is applied to the ship to make it sink to a deeper draught.

Example 3.4

A box 2m x 1m (LxB) in sea water is floating at a draught of 0.3m.


Calculate its , , CB, CWP and TPC.

i. = L x B x T = 2 x 1 x 0.3 = 0.6m3
ii.  = L x B x T x  =  x  = 0.6 x 1.025 = 0.615 tonnes

iii. CB =  = 0.6 = 1.00


LxBxT 0.6

iv. CWP = Awp = 2x1 = 1.00


LxB 2x1

v. TPC = Awp x  = 2 x 1 x 1.025 = 0.0205


100 100

Exercise 3.7
Calculate the particulars at draught of 0.4, 0.5, 0.6 and 0.7m.

Exercise 3.8
Find hydrostatic particulars in sea water (, ,Awp,LCB, LCF,TPC) of a box barge
with dimension L=100m, B=20m, at draughts of 1.0m, 3.0m, 5.0m, 7.0m, 9.0m. If
the barge weighs 2300 tonne, what is its draught? If the barge is floating at a
draught of 4m, what is its CB?

© Omar bin Yaakob, July 2008 14


Naval Architecture Notes

It can be seen from Exercise 3.8 that for a box-shaped object at different draughts,
the waterplane areas are constant. Hence, many hydrostatics particulars remain
constant.

Exercise 3.9:

An empty cylindrical shaped tank is floating in sea water (density 1.025 t/m3) at a
draught of 8.0 m with its axis vertical. The external diameter of the tank is 12.0 m,
internal diameter 11.0 m, thickness of base 1.0 m and the overall height is 16.0 meter.
Its centre of gravity is 6 meter above its inner base.
Calculate:
.
i. Find Hydrostatic particulars , Awp, LCB, Cb, Cp, TPC, WSA
at T=1, 2, 4, 6, 8m.
ii. Plot hydrostatic curves similar to page 19 showing all data.

iii. Final draught of the tank after 500 m3 diesel oil (density 850 kg/m3) is poured
into the tank.

D 4
The second moment of area of a circle about its diameter is .
64

3.7 Hydrostatic Particulars of a Ship

Hydrostatic particulars of a real ship will be different. Consider the ship whose lines
plan is shown below. At different draughts, the ship will have different waterplane
areas, volumes and centroids. Hence, the hydrostatic particularly will vary as the
draughts changes.

© Omar bin Yaakob, July 2008 15


Naval Architecture Notes

If areas, volumes, moments, centroids of the waterplanes and sections of the ships
can be calculated, hydrostatic particulars of a ship can be obtained. These are
calculated at the design stage, once the shape and size of the ship has been
decided.

Exercise 3.10

A ship with length 100m, breadth 22m has the following volumes and areas at
different waterlines. Calculate its , CB, CW and TPC in saltwater density
1.025tonnes/m3.

Draught Aw 
(m)
2
(m )  (m3) (tonnes)
Cb Cw TPC

 Aw Aw x ro
x ro (LB) 100
LBT
2 1800.0 3168.0

4 2000.0 6547.2

6 2100.0 10137.6

8 2120.0 13728.0

10 2130.0 17424.0

The particulars can be presented in two forms, either as a set of curves or in tabular
format. Table 3.1 shows a typical table of hydrostatic particulars while an example
of hydrostatic curves is shown on page 18.

Table 3.1 Hydrostatic Particulars of Bunga Kintan LBP 100m

LCB
Draught Displacement KB BMT BML MCTC LCF
Cb (m from
(m) (tones) (m) (m) (m) (tonne-m) (m from )
)
8.00 14820.00 0.72 4.07 3.66 180.00 190.00 2.50 2.00
7.50 13140.00 0.71 3.67 3.98 195.00 183.00 2.30 1.50
7.00 11480.00 0.70 3.26 4.46 219.00 180.00 2.00 0.70
6.50 9870.00 0.69 2.85 5.02 244.00 172.00 1.80 -0.06
6.00 8280.00 0.67 2.44 5.66 279.00 165.00 1.50 -1.00
5.50 6730.00 0.66 2.04 6.67 327.00 157.00 1.10 -2.00
5.00 5220.00 0.64 1.63 8.06 392.00 146.00 0.00 -3.00

© Omar bin Yaakob, July 2008 16


Naval Architecture Notes

3.8 Using Hydrostatic Curves and Tables


Hydrostatic curves and tables can be used to obtain all hydrostatic particulars of a
ship once the draught or any one of the particulars is known.

Example 3.5

From MV Bulker hydrostatic Curves (pg18) at a draught of 7m, we can obtain


displacement  = 31,000 tonnes, LCF = 2.0m forward of amidships and MCTC =
465 tonne-m etc. Also if we know the ship weighs 40,000 tonnes, its draught, TPC,
MCTC, LCF and LCB can be obtained.

Exercise 3.11

Using MV Bulker Hydrostatic Curves, find displacement, LCB, LCF, TPC at draught
of 9.5m. If 1500 tonnes is added to the ship, what is its new draught?

Hydrostatic tables can be used in a similar manner to obtain hydrostatic particulars


once draught is known or to obtain draught and other particulars once the
displacement or another particular is known. There is however a need to interpolate
the table to obtain intermediate values.

HOMEWORK 1:

By using the hydrostatic particulars of Bunga Kintan shown in Table 3.1:

i. Draw full hydrostatic curves of the ship


ii. Find values of displacement , KB , LCB, BMT, BML, MCTC, CB,
LCF of the ship if it is floating at a draught of 6.75m.
iii. Find values of T, KB , LCB, BMT, BML, MCTC, CB, LCF of ship if
the ship weighs 11,480 tonnes.
iv. When the ship is floating at a draught of 5.5m, 3000 tonne cargo
was added. What is its new draught?

Submission Date: _______________________

Exercise 3.11:

Calculate , , KB , LCB, Aw, TPC, CB, CP, CM, CW, LCF of a cylinder radius 1m
floating with axis vertical at draughts of 1.0, 1.5, 2.0 and 2.5m.

© Omar bin Yaakob, July 2008 17


Naval Architecture Notes

MV Bulker Hyrostatic Curves

© Omar bin Yaakob, July 2008 18


Naval Architecture Notes

Chapter 4
Basic Stability Consideration
4.1 Introduction

One of the factor threatening the safety of the ship, cargo and crew is the lost or
lack of stability of the vessel. Stability calculation is an important step in the design
of the ship and during its operation. While designing the ship, the designers must
be able to estimate or calculate to check whether the ship will be stable when
constructed and ready to operate. For the ship's master, he must be able to load
and stow cargo and handle the ship while ensuring that the ship will be stable and
safe.

4.2 What is stability?

Stability is the tendency or ability of the ship to return to upright when displaced
from the upright position. A ship with a strong tendency to return to upright is
regarded as a stable vessel. On the other hand, a vessel is said to be not stable
when it has little or no ability to return to the upright condition. In fact, an unstable
ship may require just a small external force or moment to cause it to capsize.

Figure
4.1

An
(a) (b) (c) analog
y for
stability is often given of the marble. In Figure 1 (a), the marble in the bowl will
return to its original position at the bottom of the bowl is it is moved to the left or
the right. This marble is in a condition called positively stable. A slight push on
the marble which is put on an upside down bowl as in Figure 1 (b) will cause it to
roll off, a condition equivalent to instability. A neutrally stable ship is analogous to
a marble put on a flat surface, it will neither return nor roll any further.

4.3 Longitudinal and Transverse Stability

Ship initial stability can be seen from two aspects, longitudinally and transversely.

From longitudinal viewpoint, the effect of internal and external moments on ship's
trim is considered. Important parameters to be calculated are trim and the final
draughts at the perpendiculars of the ship. In any state, there is a definite
relationship between trim, draughts and the respective locations of the centres of
buoyancy and centre of gravity. The trim angle  is rarely taken into consideration.
Transverse stability calculation considers the ship stability in the port and
starboard direction. We are interested in the behaviour of the ship when external
statical moment is applied such as due to wind, waves or a fishing net hanging from
the side. The effect of internally generated moment such as movement of masses
on-board transversely is also studied. An important relationship considered is that

© Omar bin Yaakob, July 2008 19


Naval Architecture Notes
between heeling and righting moments and the resulting angle of heel  and its
consequence on the safety of the boat.

This Chapter will focus on basic transverse stability particularly the relationships
between the metacentre and the centre of gravity.

4.4 Basic Initial Stability: The role of GM

w
MT MT

G
W W
w G L w1 L1

W B
B L

K B1
W
K

(a) (b)

Figure 4.2

Consider the ship floats upright in equilibrium as in the above figure 4.2 (a). The
weight of the ship equals its displacement and the centre of buoyancy is directly
below the centre of gravity. When the ship is slightly disturbed from upright, the
centre of buoyancy being centre of underwater volume moves to the right. The line
of action of buoyancy vertically upward crosses the original centreline at the
metacentre, M. Since G does not move, a moment is generated to turn the ship back
to its original position. This moment is called the returning moment.

In this case, M was originally above G and we can see that the returning moment is
positive. If M was below G i.e. GM negative, the returning moment will be negative
hence the ship is unstable. If M is at G, then the ship is neutrally stable.

Righting moment is the real indication of stability i.e. the ability of the ship to
return to oppose any capsizing moment and return the ship to upright position.

The larger the righting moment, the better stability is.

© Omar bin Yaakob, July 2008 20


Naval Architecture Notes

Consider the triangle shown below:

Righting moment =  x GZ
 MT
and GZ = GMT sin 

For any displacement, righting moment depends


on GZ.

And GZ depends on GM. The bigger GZ, the


G  Z bigger Righting Moment.

 

MT

G
Relationships between K, B, G and MT are important.

KMT = KB + BMT K
KMT = KG + GMT

For any particular draught or displacement at low angle of heel, keel K and
metacentre M are fixed. Therefore the values of KB, BM and hence KM are fixed,
as can be obtained from hydrostatic particulars. Therefore the distance GMT will
only depend on the height of centre of gravity. In other words, to ensure a large
GMT, we can only ‘control’ KG.

4.5 Determining Centre of Gravity, Areas or Volumes of Composite Bodies

The above section has shown that the relative position of M and G are important in
determining ship stability. Since M is constant for any particular draught, only G
will finally determine the value of GM.

Before we go into the details of stability calculations, we have to consider how to


determine the location of G. Consider a composite body consisting of two portions
shown in Figure 4.3.
Area A

Area B
 ca C  cb

xb
xa
X
Figure 4.3

© Omar bin Yaakob, July 2008 21


Naval Architecture Notes

Distance of Centre of Composite to the reference axis:

X = A x xa + B x xb
A+B
i.e.,

X = Total moment of area about the reference axis


Total area

If the composite consists of volumes,

Centre of Volume
X = Total moment of volume about the reference axis
Total volume

If the composite consists of weights,

Centre of Gravity
X = Total moment of weight about the reference axis
Total weight

Example 4.1

Find centre of area (from AP) for an object consisting of four components shown in
the figure below.

Area Distance from AP Moment of Area


Component
(m2) (m) about AP (m3)
1 -2.5
2
3
4
TOTAL

Centroid from AP = Total moment of area about AP


Total area
= m

© Omar bin Yaakob, July 2008 22


Naval Architecture Notes
Example 4.2

A trimaran has three hulls and the respective volume displacements, LCB and KB
are shown below. Find the total displacement, LCB and KB.

Hull Volume Lcb (m aft of Kb


Displacemen amidships) (m above keel)
t (m3)
Side 1 158.7 13.0 2.5
Main 1045.8 2.0 2.0
Side 2 158.7 13.0 2.5

Hull Volume lcb (m aft Moment about Kb Moment


Displace of amidships (m4) (m above about keel
ment amidships) keel) (m4)
(m3)
Side 1 158.7 13.0 2.5
Main 1045.8 2.0 2.0
Side 2 158.7 13.0 2.5
TOTAL

LCB= Total moment about amidships = 4.56 m aft of amidships


Total Volume

KB= Total moment about keel = 2.12 m


Total Volume

Example 4.3

A stack of weights consists of one 3kg weight and two 2kg weights. Find centre of
gravity of the stack above the floor:

© Omar bin Yaakob, July 2008 23


Naval Architecture Notes
Item Weight (kg) CG above floor Moment about Floor
(cm) (kgcm)
Wt A
Wt B
Wt C
JUMLAH

Final CG = = cm

Example 4.4

A ship has three parts and the respective weights and Kg are as follows. Find the
total weight and KG.

Part Weight Kg
(tonnes) (m above keel)
Lightship 2000 5.5
Cargo 1 300 7.6
Cargo 2 500 2.5

Part Weight Kg Moment


(tonnes) (m above about keel
keel) (tonne-m)
Lightshi 2000 5.5
p
Cargo 1 300 7.6
Cargo 2 500 2.5
TOTAL

KG = Total moment about Keel = m


Total weight

Example 4.5

A ship of 6,000 tonnes displacement has KG = 6 m and KM = 7.33 m. The following


cargo is loaded:
1000 tonnes, Kg 2.5 m
500 tonnes, Kg 3.5 m
750 tonnes, Kg 9.0 m

The following cargo is then discharged:

450 tonnes of cargo Kg 0.6 m


And 800 tonnes of cargo Kg 3.0 m

Find the final GM.

© Omar bin Yaakob, July 2008 24


Naval Architecture Notes

Item Weight (tonne) Kg Moment about keel


(tonne-m)

Ship 6000 6.0


Loaded 1000 2.5
Cargo1 500 3.5
Cargo2 750 9.0
Cargo3
Unloaded -450 0.6
Cargo -800 3.0

Final moment
Final KG = Final displacement
= ________

Final KG = m

Final KM = m
Final KG = m
Ans. Final GM = m

Homework 2

A box-shaped barge is floating in sea water at a draught of 5m. The extreme


dimensions of the barge (L x B x D) are 12m x 11m x 10m. The wall and floor are 0.5m
thick. Its centre of gravity is 4m above keel.

Calculate:

i. The displacement and GMT of the empty barge.


ii. The barge is to be used to carry mud (density1500 kg/m3). If the draught of the
barge cannot exceed 7.5m, find the maximum volume of mud that can be loaded
into the barge.
iii. For the barge loaded as in (ii), find its GMT.

Submission Date: _______________

© Omar bin Yaakob, July 2008 25


Naval Architecture Notes
4.6 Movement of Centre of Areas, Volumes and Weights

When a portion is added or removed from an object, its centre moves.

Consider a homogenous object as shown below:

i. If weight is moved a distance y:

Centre of gravity moved x = GG’ = m x y


M
i.e. total moment divided by total weight

ii. If weight m is removed:

The remaining weight M-m

Movement of centre of gravity x = GG’ = m x d


M-m

i.e. total moment divided by remaining weight.

Example 4.6

A ship weighing 7000 tonnes is floating at the wharf. At that time, KM = 6.5 m and
GM 0.5m. Find new GM when a 30 tonnes box is loaded at Kg 10.0m. Assume no
change in KM.

Method 1:

Find rise in KG

Original KG = KM - GM = m
MG’
Distance 30 tonnes box from original G = m
GG’= 30 x 4.0 = 0.017m
G
10
7030
B m
KG’= KG+ GG’ = m
K
KM does not change, therefore, GM = = m

© Omar bin Yaakob, July 2008 26


Naval Architecture Notes

Method 2:

Find final KG using table of moment about keel

Portion Mass (m) Kg (m) Moment about keel


(tonne-m)
Ori. Ship 7000 6.0
Box 30 10.0
Total 7030

KG = Sum of moment
Sum of weight
KG = m

GM = KM - KG
KM - KG = m

4.7 Hanging Weights, The Use Of Derricks And Cranes

The use of cranes and derricks will make the weights suspended. Suspended
weights acts at the point of suspension. Therefore a weight that was initially
located on the lower deck for example will instantly be transferred to the point of
suspension at the instant the weight is lifted by the derrick. The centre of gravity
KG will suddenly increase and since KM is constant, GM will reduce suddenly. If
the rise in KG is more than the original GM, the net GM will be negative, leading to
instability.

Example 4.7

A ship of 7,500 tonnes displacement is upright and has GM 0.20m and KM 6.5 m.
A heavy cargo of 100 tonnes already on the lower deck (kg=2m) is to be unloaded
using the ship’s crane. When lifting the cargo crane head is 15 m above keel.
What is GM during lifting. Comment of the safety of the operation.

Treat as if the weight is suddenly transferred from lower deck to the point of
suspension, a distance of 15 meters. The KG will rise, and since KM constant, GM
will be reduced.

Original KG = KM-GM= 6.5 – 0.2 = 6.3m

Rise in KG = 100 x 13
7,500

=0.173m

KG during lifting = KG + Rise = 6.473m


GM during lifting = KM- Kgnew = 6.5- 6.473 = 0.027m

© Omar bin Yaakob, July 2008 27


Naval Architecture Notes

4.8 Free Surface Correction

When free surface exists on board the ship, stability of ship is affected. The free
surface gives rise to free surface moment which in effect reduce GM. The reduction
is called Free Surface Correction (F.S.C).

FSC is calculated from the second moment of area of the surface of the fluid;

FSC = Free surface moment


Ship displacement

Free Surface Moment (FSM) = i x ρ fluid

Where i the second moment of area of the surface of the fluid and ρ fluid is the
density of the fluid being considered.

Once the FSC is known, the new reduced GM called GMfluid is obtained

GM fluid = GMsolid - FSC

It is important that free surface be avoided or at least minimised.

Note also that KG in ships having free surface is called KG fluid and regarded
increased by FSC.

KGfluid = KGsolid + FSC

For tanks with a rectangular surface:

Free surface moment = 1 x tank length x tank breadth3 x density of fluid


12

Free surface correction = 1 x tank length x tank breadth3 x density of fluid


12 ship displacement

EXERCISE 4

1. Bunga Kintan (Hydrostatic data given on page 12) is floating at draught of


6.5m. If its KG is 6.8m, what is its GM?

2. A ship has a displacement of 1,800 tonnes and KG = 3m. She loads 3,400
tonnes of cargo (KG = 2.5 m) and 400 tonnes of bunkers (KG = 5.0m). Find
the final KG. 2.84m

3. A ship sails with displacement 3,420 tonnes and KG = 3.75 m. During the
voyage bunkers were consumed as follows: 66 tonnes (KG = 0.45 m) and 64
tonnes (KG =1 m). Find the KG at the end of the voyage.

© Omar bin Yaakob, July 2008 28


Naval Architecture Notes
4. A ship has displacement 2,000 tonnes and KG = 4m. She loads 1,500 tonnes
of cargo (KG = 6m), 3,500 tonnes of cargo (KG = 5m), and 1,520 tonnes of
bunkers (KG = 1m). She then discharges 2,000 tonnes of cargo (KG = 2.5 m)
and consumes 900 tonnes of oil fuel (KG = 0.5 m.) during the voyage. If KM=
5.5m, find the final GM on arrival at the port of destination.

5. A ship arrives in port with displacement 6,000 tonnes and KG 6 m. She then
discharges and loads the following quantities:
Discharge 1250 tonnes of cargo KG 4.5 metres
675 tonnes of cargo KG 3.5 metres
420 tonnes of cargo KG 9.0 metres
Load 980 tonnes of cargo KG 4.25 metres
550 tonnes of cargo KG 6.0 metres
700 tonnes of bunkers KG 1.0 metre
70 tonnes of FW KG 12.0 metres
During the stay in port 30 tonnes of oil (KG 1 m.) are consumed. If the final
KM is 6.8 m., find the GM on departure.

6. A ship of 9,500 tonnes displacement has KM 9.5 m and KG 9.3 m. A load


300 tonnes on the lower deck (Kg 0.6 m) is lifted to the upper deck (Kg 11 m).
Find the final GM.

7. A ship of 4,515 tonnes displacement is upright and has KG 5.4 m and KM


5.5 m. It is required to increase GM to 0.25m. A weight of 50 tonnes is to be
shifted vertically for this purpose. Find the height through which it must be
shifted.

8. A ship of 7,500 tonnes displacement has KG 5.8 m. and GM 0.5 m. A weight


of 50 tonnes is added to the ship, location Kg = 11m and 7m from centreline
to the starboard side. Find final location of G above keel and from the
centreline. What is its new GM?

9. A ship has a displacement of 3,200 tonnes (KG = 3 m. and KM = 5.5 m.). She
then loads 5,200 tonnes of cargo (KG = 5.2 m.). Find how much deck cargo
having a KG = 10 m. may now be loaded if the ship is to complete loading
with a positive GM of 0.3 metres.

10. A ship of 4,500 tonnes displacement is upright and has KG 5.4 m and KM
5.5 m. It is required to move a weight of 50 tonnes already on the deck
(kg=6m) using the ship’s derrick. The derrick head is 13 m above keel. Is it
safe to do so?
11. A ship of 9,500 tonnes displacement and has KM 9.5 m and KG 9.3 m. The
ship has two fuel tanks in double bottoms, rectangular shape each 20 x 5m
containing bunker density 900 kg/m3. Find GMfluid when free surface exists
in the tank.
12. Find Gmfluid for the ship in question 11 but with one tank only, length 20m
breadth 10m.
13. What happens to i when there are three tanks with b = 3.33m in question
11.

© Omar bin Yaakob, July 2008 29


Naval Architecture Notes
Appendix A

Second Moments of Areas

The second moment of an element of an area about an axis is equal to the product
of the area and the square of its distance from the axis. Let dA in Figure A.1
represent an element of an area and let y be its distance from the axis AB
dA

A B
Fig. A.1

The second moment of the element about AB is equal to dA x y2

2. To find the second moment of a rectangle about an axis parallel to one of its
sides and passing through the centroid

dx

x
b
G
A B

Fig. A.2

In Figure A.2, l represents the length of the rectangle and b represents the breadth.
Let G be the centroid and let AB, an axis parallel to one of the sides, pass through
the centroid.

© Omar bin Yaakob, July 2008 30


Naval Architecture Notes
Consider the elementary strip which is shown shaded in the figure. The second
moment (i) of the strip about the axis AB is given by the equation:-

i= l dx x x2

Let I AB be the second moment of the whole rectangle about the axis AB then:-

 b/2
1AB   l. x
2
.dx
- b/2

 b/2
1AB  l 
- b/2
x 2 .dx

b / 2
x3 
 l 
 3  b / 2

lb 3
1AB 
12

3. To find the second moment of a rectangle about one of its sides.


l

dx

B
A
Fig. A.3

Consider the second moment (i) of the elementary strip shown in Figure A.3 about
the axis AB.

i= l dx x x2

© Omar bin Yaakob, July 2008 31


Naval Architecture Notes
Let IAB be the second moment of the rectangle about the axis AB. Then :-
b
1AB   l. x 2 .dx
O

b
x3 
 l 
 3 O

or

lb 3
1AB 
3

4. The Theorem of Parallel Axes

The second moment of an area about an axis through the centroid is equal to
the second moment about any other axis parallel to the first reduced by the
product of the area and the square of the perpendicular distance between the
two axes. Thus, in Figure A.4, if G represents the centroid of the area (A) and
the axis OZ is parallel to AB then:-
I OZ  I AB - Ay2

A B

O Z

Fig. A.4

© Omar bin Yaakob, July 2008 32


Naval Architecture Notes

5. Second moment of area of a circle


X

A D B

Fig. A.5

For circle, the second moment of area about an axis AB.

 D4
I AB 
64

What is IXX?

6. Applications.

Second moment of areas are used in calculations of BML and BMT :

IF
BM L 

and
IT
BM T 

Where IF is longitudinal second moment of area of the waterplane about the


centre of floatation, IT is transverse second moment of area about the centreline
and  is volume displacement.

© Omar bin Yaakob, July 2008 33


Naval Architecture Notes

EXERCISES

1. Find BML and BMT of a box shaped barge 120m x 20m x 10m floating at a
draught of 7m.
2. A cylinder of radius r = 10m is floating upright at draught of 6m in fresh
water. Find its KML and KMT.

3. A fish cage consists of a wooden platform placed on used oil drums with the
following dimensions.
6m

4m

Diameter
1m.
If the total weight of the structure is 3 tonnes, floating in sea water calculate:

i) draught
ii) KMT
iii) KML

Homework 3:

A catamaran consists of two box-shaped hulls spaced 5m apart, centreline


to centreline. Each hull measures (L x B x D) 10m x 0.5m x 1m. If its
draught is 0.3m, find its :

i)  and 
ii) KB
iii) BMT
iv) Maximum allowable KG if GM minimum is 0.2m

Submission Date: _______________

© Omar bin Yaakob, July 2008 34


Naval Architecture Notes

Chapter 4 Calculations of Ship


Hydrostatic Particulars

4.1 The Importance of Hydrostatic Particulars

In the previous chapter, we have seen the importance of knowing the hydrostatic
particulars of a vessel. If we have the hydrostatic particulars in the form of tables,
curves, or our own direct calculation, we can obtain details about the ship in any
particular condition. We can also determine or estimate what would happen when
ship condition changes such as due to addition or removal of weights.

To draw hydrostatic curves or to make the table, we need to calculate the


particulars. The hydrostatic particulars can be obtained only if we carry out
calculations of area, volumes and moment at various draughts or water plane area.
Using some known relationships, the particulars can be derived from areas,
volumes and moments,

If the body has a uniform shape, such as cuboids, cones, spheres or prisms,
calculation of areas, volumes and moments are easy. For example water plane
areas, block coefficients, TPC, MCTC, KB and LCB of such objects can be found
using simple formulae. We can easily obtain the particulars at any draught and if
necessary plot the curves.

However not all ships have simple and uniform shapes as above. In fact, most
ships have hull shapes which are varying in three directions. This makes it difficult
to calculate hydrostatic particulars.

4.2 Methods to Calculate Areas

Figure 4.1 Typical Ship Half-Breadth Plan

Consider the shape of the ship whose body plan is shown in Figure 4.1. If we want
to find the area of the section or water plane for example, we do not have simple
methods. Similarly to find volume displacement or LCF will not be easy.

If we want calculate the water plane area of the ship in Figure 4.1 at a particular
draught, we may use a few methods.

© Omar bin Yaakob, July 2008 35


Naval Architecture Notes

First is to plot the curve on a graph paper from where the area under the curve can
be obtained by counting the squares. To improve accuracy, smaller boxes or
triangles can be used. The method is tedious and it’s accuracy depends on the size
of the smallest grid. To use this method, we need to plot the curve first; a
disadvantage when sometimes we are only provided with offset data, i.e. half-
breadth at various stations.

The second method is to use an equipment called the planimeter. This equipment
can be used to measure the area of a shape drawn on paper. Again, this equipment
can only be used only when hard copy of the waterline drawing is available.
Moreover, similar to graphical method, planimeter requires a lot of man power.

Figure 3.2 A Planimeter

The third method is to use mathematical approximation. In this method, an


attempt is made to represent the curve or shape by a mathematical expression. By
using calculus, area and moments of the area bounded by the curve can be found
by integration.

Mathematical methods are normally preferred for a number of reasons. First there
is no need for a hard copy of the curves. Offset tables are normally available and
the data can be used directly in the calculations. A very important feature of
mathematical methods is the ability to make use of the technology offered by
computers. The use of mathematical methods also enable us to obtain not only
areas but all hydrostatic particulars. As we have seen in chapter 3, we need to
calculate not only areas but also volumes, positions of centroids of waterplanes
(LCF) and centroids of volumes (KB and LCB). In addition we require second
moments of areas for calculations of MCTC and metacentric heights. Unlike
graphical or planimeter methods, mathematical methods can easily be used to
calculate these particulars.

A very important caution should be noted when using mathematical methods. The
accuracy of the calculations will mainly depend on the degree of fit of the actual
curve to the mathematical expression representing it.

4.3 Mathematical Methods

offset
© Omar bin Yaakob, July 2008
h 36

ST1 ST2 ST3 ST4 ST5


Naval Architecture Notes

Figure 4.3 Waterline or Sectional Area Curve

Figure 4.3 shows a curve which may represent a half-waterplane area or a curve of
sectional areas. A waterplane curve is represented by offsets made up of half-
breadth at various stations. Stations are positions along the length of the ship and
normally separated by a common-interval, h. To cater for the fast changing slopes
of the curve at the stern and bow regions, half stations may be used.

To calculate the area, centroid and moment under such curve, its offsets and h are
required. By assuming that the curve can be represented by a certain
mathematical formulae, calculations can be made. A number of methods have been
developed for these purpose such as Newton-Cottes, Tchebycheff, Trapezoidal and
Simpson methods. In this course, we will concentrate on the two most popular
methods; Trapezoidal and Simpson methods.

4.4 Trapezoidal Method


When a curve can be assumed to be represented by a set of trapezoids, the area
under the curve can be calculated.

D E
C A

B
y1 y2 y3 y4
h
A F

Figure 4.4 Waterline or Sectional Area Curve

In Figure 4.4, the area under the curve is the are area of trapezoid ABCDEF.

Area =
1
 y1  y 2 h  1  y 2  y3 h  1  y3  y 4 
2 2 2

h y1  2 y 2  2 y3  y 4 
1
=
2

Exercise

1. Find the Trapezoidal formulae for curves made up of


© Omar bin Yaakob, July 2008 37
Naval Architecture Notes
i) 6 offsets
ii) 9 offsets
iii) n offsets

2. The midship section of a chine vessel has the following offsets:

Draught(m) 0 0.25 0.5 0.75 1.0


Half-breadth (m) 0 0.6 1.0 1.5 1.9

Calculate its midship section coefficient at draught of 1.00m.

3. Find the water plane area of a ship LBP = 10m made up of the following offsets:

Station 0 1 2 3 4
Half-breadth 0 0.3 1.0 1.2 1.1
(m)

Find its area, TPC and waterplane coefficient

4.5 Simpson Rules for Areas.

Simpson rule is the most popular method being used in ship calculations to
calculate volumes, second moments of areas and centroid. This is because it is
flexible, easy to use and its mathematical basis is easily understood.

Basically, the rule states that the ship waterlines or sectional area curves can be
represented by polynomials. By using calculus, the areas, volumes, centroids and
moments can be calculated. Since the separation between stations are constant,
the calculus has been simplified by using multiplying factors or multipliers.

There are three Simpson rules, depending on the number and locations of the
offsets.

4.5.1 Simpson First Rule

Simpson’s First Rule


D
C

B
y2 y3
y1
-h h
A O E
A
Figure 4.5 Waterline or Sectional Area Curve with Three Offsets

© Omar bin Yaakob, July 2008 38


Naval Architecture Notes
Assume that the offsets are points on a polynomial curve of form

y  a0  a1 x  a2 x 2  a3 x 3

h
Then area ABCDE =

h
y.x

= 2a0 h+ 2a2h3
3

At x= -h y1 = a0 - a1h + a2h2 - a3h3


At x= 0 y2 = a0
At x= h y3 = a0 + a1h + a2h2 + a3h3

y1  y3  2 y 2
Therefore, a0  y 2 a2 
2h 2

Substituting these values into the above equation

Area ABCD =
h
y1  4 y 2  y3 
3

First Rule is used when there is an odd number of offsets. The basic multiplier for
three offsets are 1,4,1. For more stations, the multipliers are developed as follows:

Station 1 2 3 4 5 6 7

Offset a1 a2 a3 a4 a5 a6 a7

Multiplier 1 4 1
1 4 1
1 4 1
1 4 2 4 2 4 1

Area = 1/3 x h x  (multiplier x offset)

Where h = common interval

Exercise

4. Use Simpson first rule to find the area of the midship section of the chine vessel
in exercise 2. Explain the difference in area.

© Omar bin Yaakob, July 2008 39


Naval Architecture Notes

© Omar bin Yaakob, July 2008 40


Naval Architecture Notes

Example 1

Find the waterplane coefficient for the waterplane of a 27m LBP boat represented by
the following offsets:

Station 0 1 2 3 4 5 6
Half-breadth (m) 1.1 2.7 4.0 5.1 6.1 6.9 7.7

Simpson Product
Offset
Station Multiplier Area
0 1.1 1
1 2.7 4
2 4.0 2
3 5.1 4
4 6.1 2
5 6.9 4
6 7.7 1

. . Area= 1/3 x h x ∑ Product Area = ___________ m2

Cw = _______

4.5.2 Simpson Second Rules

D E
C A

B
y1 y2 y3 y4
h
A F
-3h -h 0 h 3h
2 2 2 2

Figure 4.6 Waterline or Sectional Area Curve with Four offsets

Assume that the offsets are points on a polynomial curve of form

y  a0  a1 x  a2 x 2  a3 x 3

© Omar bin Yaakob, July 2008 41


Naval Architecture Notes

Then area ABCD = 3h / 2

 y.x
3 h / 2
=  a1 x 2 a 2 x 3 a3 x 4 
a 0    
 2 3 4 

= 3 (1)
3a0  a2 h 2
4
But

3h 3a1 h 9a 2 h 2 27a3 h 3 (2)


x y1  a0   
2 2 4 8

h a1 h a 2 h 2 a3 h 3
x y 2  a0   
2 2 4 8
a1 h a 2 h 2 a3 h 3
h y3  a0   
x 2 4 8
x
3h
2 3a1 h 9a 2 h 2 27a3 h 3
y 4  a0   
2 2 4 8
Adding (2) and (4)

a2 h 2
2a 0   y1  y 2
2
Adding (2) and (5)

9a 2 h 2
2a 0   y1  y 4
2


4a2 h 2   y1  y 4   y 2  y3 
y 1  y 4  y 2  y3
a2 
4h 2

y 2  y3 a2 h 2
a0  
2 2
9 y 2 9 y 3 y1 y 4
   
16 16 10 10

h y1  3 y 2  3 y3  y 4 
3
Then area ABCD =
8

The basic multipliers are thus 1,3,3,1 and Area = 3/8 x h x  (multiplier x offset)

The rule can only be used when number of offsets = 3N +1

© Omar bin Yaakob, July 2008 42


Naval Architecture Notes

4.5.3 Simpson Third Rule

Simpson third rule is used when we have three offsets and we require the area
between two of the offsets.

Example 2
7.02

5.98

1.06
1.06

Figure 4.7 Midship Section Curve with Three offsets

A midship section curve has halfbreadth 1.06, 5.98 and 7.02 m spaced at 9.0m
draught interval. Find the area between the first two draughts.

½ Breadth Multiplier Product

1.06 5 5.30
5.98 8 47.84
7.02 -1 -7.02
46.12

Area= 1/12 x 9 x 46.12 x 2= 69.18 m2

If we require the area between two upper draughts, the calculations are as follows:

½ Breadth Multiplier Product

7.02 5 35.10
5.98 8 47.84
1.06 -1 -1.06
81.88

Area = 1/12 x 9 x 81.88 x 2 = 122.82 m2

Total Area = 192.0 m2

Exercise

5. Find the total area under the curve using Simpson first rule and compare.

© Omar bin Yaakob, July 2008 43


Naval Architecture Notes

5.6 Obtaining Volume

Volumes and hence displacement of a ship at any draught can be calculated if we


know either:

i) Waterplane areas at various waterlines up to the required draught


ii) Sectional areas up to the required draught at various stations.

Example 3

Sectional areas of a 180m LBP ship up to 5m draught in sea water at constant


interval along the length are as follows. Find its volume displacement, mass
displacement and prismatic coefficient.

Station 0 1 2 3 4 5 6 7 8 9 10
Area 5 118 233 291 303 304 304 302 283 171 0
(m2)

Figure 3.8 Section Shapes at


Various Stations

Simpso
n Produc
Statio Section Area
Multipli t Vol
n
er
0 5
1 118
2 233
3 291
4 303
5 304
6 304
7 302
8 283
9 171

© Omar bin Yaakob, July 2008 44


Naval Architecture Notes

10 0
∑f vol

1
Volume =  h   fvol
3

= ____________ m3

Similarly if we have waterplane areas, we can use Simpson rules to integrate the
areas to obtain volume. In this case the common interval is the waterline spacing.

4.7 Considering Half and Quarter Stations

Rapidly changing curvature at both ends of the ship necessitates the use of half and
quarter stations. To take this into consideration, Simpson Multipliers are also
divided as follows:

ST 1 ½ 1 1½ 2 3…

Figure 4.9 Shape with Half Stations

If we consider full stations only:

1 4 2 4…

If we consider half stations:

1 4 1
2 2 2
1 4 1
2 2 2
1 4 1

1/ 2 1 2 1 1/2 4 1
2

Example 4

A waterplane for a 120 m LBP ship has the following offsets:

Station 0 1/4 1/2 3/4 1 2 3 4 5 51/2 6


1/2 ord 0.6 2.8 4.0 5.2 6.2 9.0 9.8 8.4 4.8 2.2 0.0

Find the waterplane area, waterplane coefficient and TPC for


the waterplane.

Station 0 1/
4
1/
2
3/
4 1 2 3 4 5 51/2 6

© Omar bin Yaakob, July 2008 45


Naval Architecture Notes

Multiplie 1/4 1 1/4 1 4 1 1/2 2 1/2


r
1/ 1 1/ 1 4 1
4 4
1/ 1 1/ 1 11/4 4 2 4 11/2 2 1/
4 2 2

© Omar bin Yaakob, July 2008 46


Naval Architecture Notes

Statio 1/2 SM Product Area Lever Product Product


n ord 1st moment 2nd mmt

0 0.6 1/
4

1/4 2.8 1
1/2 4.0 1/2

3/4 5.2 1

1 6.2 11/4
2 9.0 4

3 9.8 2

4 8.4 4

5 4.8 11/2

5.5 2.2 2

6 0.0 1/2

. .Area= 1/3 x h x ∑ Product Area = ___________ m2

Cw = _______

TPC = ________

4.8 Obtaining LCF, LCB and Longitudinal Second moment of Area

dx

x
A

© Omar bin Yaakob, July 2008 47


Naval Architecture Notes

Area =  ydx => 1/3 x productA x h


1st moment =  ydx x
=  x . ydx => 1/3 x product1stmmt x h x h

2st moment = y dx x 2

= x => 1/3 x product2ndmmt x h x h2


2
ydx

Values of x are given in multiples of h, the common interval.

If the product for area is multiplied by multiples of h, called levers, the sum of
products can be used to find the first moment and hence the longitudinal position
of the centroid.

1
LCF =  h  h   product 1st moment
3
1
 h   product area
3

 product 1st moment x h


=

 product area

If the offsets are half-breadths, the centroid is LCF. If the offsets are sectional
areas, the centroid is centre of volume i.e. LCB. The LCF is measured from the axis
where levers are taken.

For second moment, Simpson’s product for areas are multiplied twice with levers.
Again, the second moment are taken about the axis from where levers are taken.

1
IL   h  h  h   product 2nd moment
3

Example 5

Find the area, LCF , second moment of area about amidships, transverse second
moment of area about centreline for the waterplane of a ship LBP 180m with the
following ordinates.

Stesen AP 1/2 1 2 3 4 5 6 7 8 9 91/2 FP


1/2ord (m) 0 5 8 10.5 12.5 13.5 13.5 12.5 11.0 7.5 3.0 1.0 0

© Omar bin Yaakob, July 2008 48


Naval Architecture Notes

½
Product Product Product
Station ordinat SM Lever Lever
Area 1stmmt 2ndmmt
e
AP 0 ½ - +5 - +5 -
½ 5.0 2 10.0 +4 ½ +45.0 4½ +202.5
1 8.0 1½ 12.0 +4 +48.0 +4 +192.0
2 10.5 4 42.0 +3 +126.0 +3 +378.0
3 12.5 2 25.0 +2 +50.0 +2 +100.0
4 13.5 4 54.0 +1 +54.0 +1 +54.0
5 13.5 2 27.0 0 Sum_aft 0 -
+323.0
6 12.5 4 50.0 -1 -50.0 -1 +50.0
7 11.0 2 22.0 -2 -44.0 -2 +88.0
8 7.5 4 30.0 -3 -90.0 -3 +270.0
9 3.0 1½ 4.5 -4 -18.0 -4 +72.0
9½ 1.0 2 2.0 -4 ½ -9.0 -4½ +40.5
FP 0 ½ - -5 - -5 -
278.5 Sum_fwd 1447.0
-211.0

1 180
Waterplane Area =   278.5  2  3342.0 m 2
3 10

 product 1st moment x h


LCF =

 product area

(323  211)  180


LCF =  7.24 m aft of amidships
278.5 10

1
IL   h  h  h   product 2nd moment
3

IL = 2 x 1 x 183x 1447.0 = 5,625, 936 m4.


3

© Omar bin Yaakob, July 2008 49


Naval Architecture Notes

Exercise 6:

1. Repeat the Example 5 but this time,


 Calculate LCF from AP and 2nd Moment of area about AP.
 Check that the answers are identical.

2. Calculate the centroid of the midship section in Example 2 measured from


the top-most waterline (page 8).
3. Calculate LCB of the vessel in on Example 3.
4. Calculate LCF from amidship and longitudinal second moment of area about
amidship of the ship in Example 4 on page 11.

4.9 Obtaining Second Moment Of Area About The Centreline

dx

x
A

If the shaded area is a rectangle, second moment of area about the x-axis is

1
i = dxy 3
3

for the whole area :


1 3
3
IT  y dx

If the ordinates are cubed and Simpson multipliers are applied,

1 1
I T    h   product 2nd moment
3 3
1
  h   product 2nd moment
9

© Omar bin Yaakob, July 2008 50


Naval Architecture Notes

Example

Find BMT for a waterplane of a ship LBP = 100m with the following half breadths.
At this draught the ship has a displacement of 11275 tonnes in sea water.

AP ½ 1 2 3 4 5 6 7 8 9 91/2 FP
0 5 8 10.5 12.5 13.5 13.5 12.5 11 7.5 3 1 0

Station ½ (½ SM Product for


ordinat ordinate)3 Second
e Moment T
AP 0 - ½ -
½ 5.0 125.0 2 250.0
1 8.0 512.0 1½ 768.0
2 10.5 1157.6 4 4630.4
3 12.5 1953.1 2 3906.2
4 13.5 2460.4 4 9841.6
5 13.5 2460.4 2 4920.8
6 12.5 1953.1 4 7812.4
7 11.0 1331.0 2 2662.0
8 7.5 421.9 4 1687.6
9 3.0 27.0 1½ 40.5
9½ 1.0 1.0 2 2.0
FP 0 - ½ -
36521.5

2nd Moment = 1/3 x 1/3 x h x product mmt x 2 =


81158.9 m4
about amidships

Volume Displacement = 11275 = 11000 m3


1.025
BMT = 81158.9 = 7.38m
11000

4.10 Appendages

Appendages are the portion of the hull which is protruding from the main body. It
may be part of underwater volume such as a skeg or keel or parts of a waterplane
area which is not suitable to be integrated with the main area due to its abrupt
change in area.

Areas, volumes and moment are calculated separately for the appendages and later
incorporated using composite body method explained in Chapter 4 of NA1 notes.

© Omar bin Yaakob, July 2008 51


Naval Architecture Notes

Example

A ship length 150m, breadth 22m has the following areas at the various draft.

Draught (m) 2 4 6 8 10
Area of 1800 2000 2130 2250 2370
Waterplane(m2 )

There is an appendage (between waterline 0 and 2m) with displacement 2600 tonne
in sea water and Kb of 1.2m. Find the total displacement, KB and Cb of the ship at
10m draught.

Solution:

Draught Aw (m2 ) Multiplier Product Lever Product


(m) for for 1st
Volume Moment
2 1800 1 1800 0 0
4 2000 4 8000 1 8000
6 2130 2 4260 2 8520
8 2250 4 9000 3 27000
10 2370 1 2370 4 9480
25,430 53,000

Volume Displacement = 1/3 x 2x 25430 = 16960 m3

Mass  = 16960 x  =17380 tonne

Centre of Buoyancy = 53000 x 2 = 4.16m above 2m WL.


25430

Composite Table

Portion Displacemen KB Moment


t
(tonnes)
Main(2m- 17380 6.16 107,000
10m)
Appendage 2600 1.20 3,120
Total 19980 110,120

KB = 110,120 = 5.51m
19980

CB = 19980 = 0.59
150 x 22 x 10 x 1.025

© Omar bin Yaakob, July 2008 52


Naval Architecture Notes

© Omar bin Yaakob, July 2008 53


Naval Architecture Notes

4.11 Simpson Rules for Radial Integration

C

A r

1 2
Strip Area = r dθ
2

1
Total Area =  2r dθ
2

1
2
= r 2d θ

In Simpson terms, if first rule is used;

1 1
Total area =   h   fA
2 3

in radians

Example

e.g. A figure is bounded by two radii at right angles to each other and a plane
curve. The polar coordinates of the curve at equal interval of angle are 10,9,8,7,6,5
and 4 meters respectively. Find the area of the figure and its centroid from the 10m
radius.

© Omar bin Yaakob, July 2008 54


Naval Architecture Notes
Angle r r2 SM Product r3 Sin r3xsinxSM
for angle
Area
0 10 100 1 100
15 9 81 4 324
30 8 64 2 128
45 7 49 4 196
60 6 36 2 72
75 5 25 4 100
90 4 16 1 16
936 3157.95

Area 1 h
   fA
2 3

1 1 90 
     936
2 3 6 180

= 40.6 Sq. metres

Centroid is measured perpendicular from one boundary

1
3
1st moment about AB = r 3 sin θ d θ

 Centroid `x moment =
area
1 1
  h   fmmt
= 3 3
1 1
  h   f
2 3

2  fmmt
= 
3  f

Centroid from 10m boundary = 2 x 3157.95 = 2.25m


3 x 936

Exercise 7

Find area bounded by a plane curve and two radii 900 apart, if the lengths of the
radii at equal angle intervals are 2,3,5,8, and 10 metres respectively. Also find the
distance of the centroid of the figure from the 2m radius.

© Omar bin Yaakob, July 2008 55


Naval Architecture Notes

4.12 Tchebycheff’s Rule

When y2 is the middle ordinate and y1 and y3 are located 0.7071l to the left and
right of y2,

Area = C (y1 +y2+y3) where C= L/number of ordinates and l is 0.5L

Ordinates are not equally spaced and their positions in the length depend on
number of ordinates, n.

n Position of ordinates from centre of length


expressed as fraction of half length
2 0.5773
3 0 0.7071
4 0.1876 0.7947
5 0 0.3745 0.9325
6 0.2666 0.4225 0.8662
10 0.0838 0.3127 0.50 0.6873 0.9162

Example

Find area of a 200m waterplane if the half breadth at Tchebycheff stations are as
follows:
1.2, 5.0, 8.4, 10.5, 11.7, 11.8, 11.1, 9.6, 7.4, 3.8

C= L/10 = 200/10
Sum of y =
Area = 3220 m2

© Omar bin Yaakob, July 2008 56


Naval Architecture Notes

EXERCISES 4

Question 1

A cargo ship 120m, breadth 25m and depth 16m is floating at 8.5m draught
in sea water. The area of sections at various stations are shown in the
following table:

Statio AP 1 2 3 4 5 6 7 8 9 FP
n
As 12.8 64.5 100. 120. 154. 166. 140. 125. 97.6 43.2 0.0
(m2) 0 6 2 8 7 9

Calculate

i. Mass Displacement
ii. Longitudinal Centre of Buoyancy (LCB) from amidships.
iii. Block Coefficient (CB)
iv. Midship Section Coefficient (CM)
v. Prismatic Coefficient (Cp)

COPYING (zero marks), UNTIDY (minus up to 1 mark)


Question 2

a. At a draught of 4m, the waterplane of Containership Bunga Bawang


(LBP=88m) has the following offsets.

Station 0(AP) 1 2 4 6 7 8(FP)

½ Breadth 2.20 4.48 6.22 7.10 5.02 2.53 0


(m)

Calculate area of waterplane, waterplane coefficient, TPC and LCF from


amidship.

b. The waterplane areas of Bunga Bawang at other draughts are as follows:

Draughts 1m 2m 3m

Area (m2) 520 690 830

Between the keel and 1m waterplane, there is an appendage with volume


420 m3 and centroid 0.60m above keel.

Use all the information to calculate for the ship at draught of 4m, the total
mass displacement in sea water, its block coefficient and centre of buoyancy
above keel.

© Omar bin Yaakob, July 2008 57


Naval Architecture Notes

Question 3

The 2m waterplane of a catamaran boat LBP 20m is shown in Figure 1. The


half-breadths of one hull is shown in the following table:

Station 0 (AP) 1 2 3 4 5
(amidships)
½ lebar 2.20 2.18 2.16 2.14 2.12 2.10
(m)

Station 6 7 8 8.5 9 9.5 10 (FP)


½ lebar 2.0 1.8 1.6 1.2 0.9 0.4 0.00
(m) 0 0 0 0 0 0

Calculate for the total waterplane:


area of waterplane, TPC, LCF and second moment of area about the
centreline
20m

CL
3m

Figure 1 Catamaran Waterplane

Question 4

A ship LBP 90m, lebar 17.2 m is floating in seawater. At 5m draught, the


waterplane has the following offsets.

Stn. AP 1 2 3 4 5 6 7 8 9 FP
½ 0.0 5.5 8.0 8.4 8.5 8.6 8.5 8.0 7.0 4.5 0.0
breadth
(m)

The ship has the following waterplane area at other draughts:

Draught (m) 0.0 0.5 1.0 2.0 3.0 4.0


Waterplane 10 500 800 1100 1200 1260
Area(m2)
© Omar bin Yaakob, July 2008 58
Naval Architecture Notes

Using all the information, calculate for draught of 5.0m:


i. Waterplane Area
ii. Second moment of area about the centreline.
iii. Mass displacement.
iv. Block Coefficient Cb
v. Height of Metacentre, KMT

© Omar bin Yaakob, July 2008 59


Naval Architecture Notes

Question 5

The cross-section of a tank can be represented by a plane curve and two radii 90 0 apart as
shown in Figure 2. The lengths of the radii at equal angle intervals are 12,14, 16,18, and 20
metres respectively. Calculate the area of the cross-section.

20m

12m

Figure 2

Question 6

a. Sebuah lengkung dinyatakan seperti berikut:


y = 2 +3x +4x2
Tentukan luas di bawah lengkung yang disempadani oleh x = 0 to x = 4
dan paksi x menggunakan kaedah:

i) Simpsons Pertama
ii) Simpsons Kedua
iii) Trapezoid
iv) Pengamiran

Berikan komentar terhadap keputusan yang diperolehi.

b. Ofset bagi sebuah kapal LBP 60m adalah seperti berikut:

Stesen 0 1 2 2 ½ 3 4 5
(AP) (FP)
Separuh
Lebar 0.5 1.4 2.6 4.3 5.4 6.6 7.0
(m)

Kirakan :
i) Luas Satahair
ii) LCF dari peminggang
iii) Momen luas kedua melintang pada garis tengah.
iv) Momen luas kedua membujur pada pusat keapungan.

© Omar bin Yaakob, July 2008 60


Naval Architecture Notes

© Omar bin Yaakob, July 2008 61


Naval Architecture Notes

Chapter 5 Transverse Stability


Consider a ship floating upright as shown in Figure 5.1. The centres of gravity and
buoyancy are on the centre line. The resultant force acting on the ship is zero, and
the resultant moment about the centre of gravity is zero.

M-

W L

G
B-

K
W

Figure. 5.1

Now let a weight already on board the ship be shifted transversely such that G
moves to G1 as in Figure 5.2. This will produce a listing moment of W X GG 1 and
the ship will start to list until GI and the centre of buoyancy are in the same vertical
line as in Figure 5.3.

M-

W L

G G1
B-

K
W
Figure. 5.2

© Omar bin Yaakob, July 2008 62


Naval Architecture Notes

M
W1 L1
G Z M
G1
B 
B1 L

G
W

Figure. 5.3 G1

In this position G1 will also lie vertically under M so long as the angle of list is small.
Therefore, if the final positions of the metacentre and the centre of gravity are
known, the final list can be found, using trigonometry, in the triangle GG1M which
is right-angled at G.

In triangle GG1M: GG1 = w x d


W

Tan  = GG1
GM

Tan  = wxd
W x GM

The formula can be restated as:

Tan  = listing moment


W x GM

It can be seen that GM plays a big role in determining angle of list. The bigger GM,
the less the angle of list and vice-versa.

The final position of the centre of gravity and hence GM is found by taking moments
about the keel and about the centre line as discussed in Chapter 4. ‘

Note. It will be found more convenient in calculations, when taking moments, to


consider the ship to be upright throughout the operation.

© Omar bin Yaakob, July 2008 63


Naval Architecture Notes

Example I

A ship of 6,000 tonnes displacement has KM = 7.3 m, and KG = 6.7 m, and is


floating upright. A weight of 60 tonnes already on board is shifted 12 m
transversely.
Find the resultant list.

Figure 5.4(a) shows the initial position of G before the weight was shifted and Figure
5.4(b) shows the final position of G after the weight has been shifted.

When the weight is shifted transversely the ship’s centre of gravity will also shift
transversely, from G to G1. The ship will then list  degrees to bring G1 vertically
under M the metacentre

GM = KM - KG = 0.6m
Listing Moment = 60 x 12 tonne-m

Tan  = 60 x 12
6000 x 0.6

Tan  = 0.2
Ans. List = 11 18 ½ ‘

W W

M- M-

W L W L

G- G G1
B- B-

K K
W W

(a) (b)
Figure. 5.4

Example 2

A ship of 8,000 tonnes displacement has KM = 8.7 m, and KG = 7.6 m The following
weights are then loaded and discharged:
 Load 250 tonnes cargo KG 6.1 m and centre of gravity 7.6 m to starboard of
the centre line.
 Load 300 tonnes fuel oil KG 0.6 m and centre of gravity 6.1 m to port of the
centre line.
© Omar bin Yaakob, July 2008 64
Naval Architecture Notes
 Discharge 50 tonnes of ballast KG 1.2 m and centre of gravity 4.6 m to port
of the centre line.
Find the final list.

Note. In this type of problem find the final KG by taking moments about the keel,
and the final listing moment by taking moments about the centre line.
(1) Moments about the keel

Weight KG Moment about


keel
8000 7.6 60800
250 6.1 1525
300 0.6 180
-50 1.2 -60
8500 62445

Final KG = Final moment KM = 8.7 m.


Final displacement Final KG = 7.34 m.

= 62.445 Final GM = 1.36 m.


8500

Final KG = 7.34 m

M- M-

50 G - 7.6 m
t G G1
4.6 m
K B-
250t
6.1 m W
K

300t
(a) (b)
Figure 5.5

Moments about the centre line

Listing moment
W .d (tonne-m)
to port to

© Omar bin Yaakob, July 2008 65


Naval Architecture Notes
starboard

250 7.6 1900


-50 4.6 -230
300 6.1 1830 -

1600 1900

Net listing moment 300


Since the final position of the centre of gravity must lie vertically under M, it follows
that the ship will list  degrees to starboard.
Tan  = Listing Moment
W x GM
= 300
8500 x 1.36
  = 1 29 ½’
Ans. Final list = 129 ½’ to starboard

Example 4

A ship of 13,750 tonnes displacement, GM = 0.75 m, is listed 2.5 degrees to


starboard and has yet to load 250 tonnes of cargo. There is space available in each
side of No.3 ‘tween deck (centre of gravity, 6.1 m out from the centre line). Find
how much cargo to load on each side if the ship is to be upright on completion of
loading.
M- 2 ½

G1
G
w
6.1 6.1
m m

w 250-w
K

Figure 5.6

Tan  = listing mmt


W x GM

1. Find listing moment that is initialy listing the ship to starboard;

listing moment = W x GM x tan  = 13750 x 0.75 x tan 2.5 = 450.25 tonne-


m S clockwise

2. Load ‘w’ tonnes to port and (250-w tonnes) to starboard.

© Omar bin Yaakob, July 2008 66


Naval Architecture Notes
Net moment of the new load (anticlockwise) must counterbalance the original
moment (clockwise):

[New moment to port]– [New moment to starboard] = 450.25


[w x 6.1] – [(250 - w) x 6.1] = 450.25
w = 161.9 tonne to port
88.1 tonne to starboard

Example 5

A ship of 9,900 tonnes displacement has KM = 7.3 m and KG = 6.4 m. She has yet
to load two 50 tonne boxes with her own gear and the first box ls to be placed on
deck on the inshore side (KG 9 m and centre of gravity 6 m out from the centre
line). When the derrick plumbs the quay its head is 15 m above the keel and 12m
out from the centre line. Calculate maximum list during operation.
Note: The maximum list is obviously occur when the first box is in place on the deck
and the second box is suspended over the quay as shown in Figure 5.7.
12m

6m

o 15m

G1 G2 50t 50t

W G- 9m
L
W

Figure 5.7

(1) Moments about the keel

weight KG Moment

9900 6.4 63,360


© Omar bin Yaakob, July 2008 67
Naval Architecture Notes
50 9.0 450
50 15.0 750
1000 64,560
0

Final KG = Final moment

Final displacement

= 64,560
10000
= 6.456m

(2) Moments about the centre line

Listing moment
W .d

to port to starboard

50 12 - 600
50 6 - 300

900

i.e. listing moment = 900 tonnes metres

(3)
New GM = 7.3- 6.456
= 0.844 m.
tan  = Listing moment
W x GM
= 900
10000 x 0.844

Ans. Maximum list = 6 6’

© Omar bin Yaakob, July 2008 68


Naval Architecture 2 Notes

Exercise 5

1. A ship of 6,000 tonnes displacement has KM = 7.3 m, and KG = 6.7 m, and


is floating at a list of 11.3 degrees to starboard. Find how much water to be
transferred from starboard to port tanks, a distance of 5 meters to bring the
ship to upright.

2. A ship of 5,000 tonnes displacement has KG 4.2 m, KM 4.5m and is listed


5 degrees to port. Assuming that KM remains constant, find the final list if
80 tonne of bunker is loaded in No 2 starboard tank whose centre of
gravity is 1 meter above the keel and 4 metre out from the centre line. (6
deg 3 min)

3. A ship of 4,515 tonnes displacement is upright and has KG 5.4 m and KM


5.8 m It is required to list the ship 2 degrees to starboard and a weight of
15 tonnes is to be shifted transversely for this purpose. Find the distance
through which it must be shifted. (4.2m)

4. A ship of 7,800 tonnes displacement has a mean draft of 6.8 m and is to be


loaded to a mean draft of 7 metres. GM 0.7 m, TPC 20 tonnes. The ship is
at present listed 4 degrees to starboard. How much more cargo can be
shipped in the port and starboard ‘tween decks, centres of gravity 6 m and
5 m respectively from the centre line, for the ship to complete loading and
finish upright. (216.5 tonnes Port, 183.5 tonnes Stb)

5. A ship of 1,500 tonnes displacement has KG 2.7 m, and KM 3.1 m and is


floating upright in salt water. Find final list if a weight of 10 tonnes is
shifted transversely across the deck through a distance of 10 metres. (9.5
deg)

6. A weight of 12 tonnes when moved transversely across the deck through a


distance of 12 m, causes a ship of 4,000 tonnes displacement to list 3.8
degrees to starboard. If KM 6 m, find the KG. (5.46m)

7. A quantity of grain, estimated at 100 tonnes, shifts 10 m horizontally and


1.5 m vertically in a ship of 9,000 tonnes displacement. If the ship’s
original GM was 0.5 m, find the resulting list. (13 deg)

8. A ship of 7,500 tonnes displacement has KM 8.6 m, KG 7.8 m and 20 m


beam. A quantity of deck cargo is lost from the starboard side (KG 12 m
and centre of gravity 6 m in from the rail). If the resulting list is 3 degrees
20 minutes to port, find how much deck cargo was lost. (XX.X tonnes)

9. A ship of 12,500 tonnes displacement, KM 7 m, KG 6.4 m, has a 3 degree


list to starboard and has yet to load 500 tonnes of cargo. There is space
available in the ‘tween decks, centres of gravity 6 m each side of the centre
line. Find how much cargo to load on each side if the ship is to complete
loading upright. (282.75 tonnes P)

10. A ship is listed 2.5 degrees to port. The displacement is 8,500 tonnes KM
5.5 m, and KG 4.6 m. The ship has yet to load a locomotive of 90 tonnes
mass on deck on the starboard side (centre of gravity 7.5 m from the centre
line), and a tender of 40 tonnes. Find how far from the centre line the
tender must be placed if the ship is to complete loading upright, and also
find the final GM. (KG of the deck cargo is 7 m.)
© Omar bin Yaakob, July 2006 69
Naval Architecture 2 Notes

11. A ship of 9,500 tonnes displacement is listed 3.5 degrees to starboard and
has KM 9.5 m and KG 9.3 m. She loads 300 tonnes of bunkers in No.3
double-bottom tank port side (KG 0.6 m and centre of gravity 6 m from the
centre line), and discharges two parcels of cargo each of 50 tonnes from the
port side of No.2 Shelter Deck (KG 11 m and centre of gravity 5 m from the
centre line). Find the final list. (14 deg)

12. A ship of 6,500 tonnes displacement is floating upright and has GM 0.15 m.
A weight of 50 tonnes. already on board, is moved 1.5 m vertically
downwards and 5m transversely to starboard. Find the list. (13 deg)

13. A ship of 5,600 tonnes displacement is floating upright and has KG 5.5 m,
and GM 0.5 m. A weight of 30 tonnes is lifted from the port side of No.2
‘tween deck to the starboard side of No.2 shelter deck (10 m horizontally
and 3 m vertically). Find the weight of water to be transferred in No.3
double-bottom tank from starboard to port to keep the ship upright. The
distance between the centres of gravity of the tanks is 6 metres.

© Omar bin Yaakob, July 2006 70


Naval Architecture 2 Notes

Chapter 6 Longitudinal Stability


6.1 Introduction

In the previous chapter, we looked at transverse stability i.e. we considered


the resultant angles of heel when heeling moments are applied to the ship. In
this chapter, we will consider stability in the longitudinal direction. In this
case we will look at the effect of change in trimming moments on the trim of
the ship and the resulting draughts at the perpendiculars.

6.2 Trim due to movement of weights

ML

G’ G
F

B’ B

Figure 6.1

Consider the ship as in Figure 6.1. If the weight w is moved a distance d


meter, G will move to G’ parallel to the direction of movement of w.

wxd
G G’ =

The shift in weight results in a trimming moment wd and the ship will trim
until G and B are in line. LCF, the centre of floatation is the centre of area
of the water plane. For small trim, the ship is assumed to be trimming about
LCF.

The trimming moment causes change in trim and hence change in draughts
at AP and FP.

Change Trim = trimming moment


MCTC
© Omar bin Yaakob, July 2006 71
Naval Architecture 2 Notes

Changes in draught forward, TF and aft, TA can be obtained by dividing
trim in proportion to the distance from LCF to the positions where the
draughts are measured, normally AP and FP.

TA x

F
TA TF trim

TF

LBP

Figure 6.2

trim = TA + TF

 LBP   LBP 
 - x  + x
TA = trim x  2  TF = trim x 
2 
 LBP   LBP 
   
Example 6.1

A ship LBP 100 m has MCTC 125 tonne-m while its LCF is 2.0 m aft of
amidships. Its original draughts are 4.5 m at AP and 4.45 m at FP.

Find new draughts when a 100 tonne weight already on board is moved 50m
aft.

Change Trim = trimming moment


MCTC

100  50
Change in trim = = 40 cm = 0.4 m by stern
125

 50  2.0 
TF  0.4   
 100 

= - 0.208 m
(since ship trims aft, forward draught is reduced i.e. negative)

© Omar bin Yaakob, July 2006 72


Naval Architecture 2 Notes

and similarly,

TA = + 0.192 m, positive due to increase in draught aft.

Final draught table;

TA TF
Original 4.5 4.45
T + 0.192 - 0.208
New 4.692 m 4.242 m

Exercise:

A ship LBP 50m is floating at Ta= 5.1m and Tf =5.3m. In this condition its MCTC is
30 tonne-m, LCF 5m fwd of amidships.

Find new draughts at Ap and FP when 50 tonne weight is moved 15m forward.

6.3 Small Weight Changes

If a small weight w is added or removed from a ship, the draught of the ship
will change as follows:

w
i. Parallel sinkage/rise
TPC

trimming moment w x distance to LCF


ii. Change in trim = =
MCTC MCTC

Once trim is obtained, the changes TF and TA can be calculated and the
final draughts will include the parallel rise/sinkage and TF & TA.

© Omar bin Yaakob, July 2006 73


Naval Architecture 2 Notes

Example 6.2

A ship LBP 100 m has LCF 3 m aft of amidships and floats at 3.2 m and 4.4
m at FP and AP respectively. Its TPC is 10 tonne while MCTC 100 tonne-m.
50 tonne cargo is removed from 20 m forward of amidships while 30 tonne is
unloaded from cargo hold 15 m aft of amidships. Find the final draughts at
the perpendiculars.

When cargo is removed, draught reduces i.e. the ship rise.

Parallel rise = 80 = 8 cm = 0.08 m


10
At the same time, the ship trims because there is a change of moment about
LCF.

Change in Moment = sum of moment of weight about LCF


= 50 x (20 + 3) – 30 x (15 – 3)
(aft) = 790 tonne-m

790
Change in trim = = 0.079 m
100
 100 
  3
TA = 0.079  2 = + 0.037 m
 100 
 
 
 100 
  3
TF = 0.079  2 = - 0.042 m
 100 
 
 

TA TF
Original 4.4 m 3.2 m
Rise - 0.08 - 0.08
T + 0.037 - 0.041
4.357 m 3.089 m

6.4 Effect of Large Changes in Mass

The effects discussed in Section 6.3 are related to small changes in weights.
These small changes lead to small changes in draughts and the hydrostatic
particulars are assumed unchanged.

If the weight changes are big, draughts will change significantly. Hydrostatic
data such as TPC and MCTC will also change and therefore the simple
formula used in Section 6.3 can no longer be used.

© Omar bin Yaakob, July 2006 74


Naval Architecture 2 Notes

G (final) G
(original)

Weight W

p Buoyancy 
B (original)

Figure 6.3 Relative Positions of G and B

Consider the relative positions of centre of gravity and centre of gravity as


shown in Figure 6.3. Before addition or removal of weight, B is directly
under G and in equilibrium, buoyancy  equals weight W.

When weights are added, the draught of the ship can be considered to
change as follows:

i.) The effect of additional weight will cause the vessel to sink to new
draught. But since there is a big change in draught, TPC cannot be
used. Instead, the mean draught of the ship must be obtained from
hydrostatic particulars. At this mean draught, also obtain LCB, MCTC
and LCF.
ii.) From the way the vessel is loaded, the final LCG can be calculated.
The new location of G is no longer directly under the new LCB of the
ship i.e. a trimming moment is created which will trim the vessel. If
the longitudinal separation between G and B is p, then trim can be
calculated:

xp
Trim (m) =
MCTC x 100

LCG, LCB, MCTC and  in this formula are for the final condition of
equilibrium.

This formula is important for two cases:

i) If we know the condition of the ship in terms of its weight and LCG as
well as its hydrostatic particulars (LCB, LCF, MCTC) we can find its
trim and the draughts at the perpendiculars.
ii) If we know ships draughts, we can find trim and hydrostatic
particulars (LCB, MCTC, LCF, displacement). Using the above
formula, we can find LCG.

Direction of trim (by stern or bow) can be derived by considering the relative
position of LCG and LCB giving the direction of the trimming moment.

G G
© Omar bin Yaakob,
) July 2006 ) 75

Naval Architecture 2 Notes

Trim by Stern Trim by Bow

Figure 6.4

6.5 Loading Calculations

Loading calculation is an important step in the ship design process.


Through this calculation, we are able to calculate the final displacement of
the ship, its KG, LCG and final draughts at the perpendiculars. Also, the
value of GM can be estimated. The calculation is done for a number of ship
loading conditions.

A loading table is used where the weights of the ship and any additional
deadweight and the respective kg and lcg are tabulated. Finally the total
weights, KG and LCG can be obtained. By using hydrostatic table, values of
mean draught, MCTC, LCF, KMT and LCB are obtained. The formula above
is used to calculate ships draughts at AP and FP. Also, since KG and KMT
are known, the value of GM can be obtained.

The above calculations are carried out while the ship is still on the drawing
board.

Loading calculation is also important for the ship's master to know the
current condition of his ship. Calculations can be done to determine the
current level of stability and draughts when some loading and unloading are
done on the ship.

To undertake loading calculations, the following items are required:

a) Hydrostatic data, table or particulars.

b) The weight, KG and LCG of the original ship whether lightship or already
loaded.

c) A list of loads to be added or removed from the ship; their masses, kg and
lcg.

Example 6.3

© Omar bin Yaakob, July 2006 76


Naval Architecture 2 Notes

A ship LBP 125m having lightship mass 4000 tonne, LCG 1.6m aft of
amidships is loaded with the following:

8500 tonne cargo lcg 3.9m forward of amidships


1200 tonne fuel lcg 3.1m aft of amidships
200 tonne water lcg 7.6m aft of amidships.
100 tonne store lcg 30.5m fwd of amidships

Hydrostatic particulars indicate that at 14000 tonne displacement, mean


draught is 7.8m, MCTC 160 tonne-m, LCB 2.00m forward of amidships and
LCF 1.5m forward of amidships. Find the final draughts at the
perpendiculars.

Item Mass LCG Fwd Moment Aft Moment


(tonnes) (m from about about
amidship amidships amidships
s) (tonne-m) (tonne-m)
Lightship 4000 1.6A 6400
Cargo 8500 3.9F 33150
Fuel 1200 3.1A 3720
Water 200 7.6A 1520
Store 100 30.5F 3050
TOTAL 14000 36200 11640

Forward Excess Moment = 36200 - 11640


= 24560 tonne m

LCG from amidships = 24560


14000
= 1.754m fwd of amidships

From hydrostatics, LCB 2.00m forward of amidships

Trim = 0.246 x 14000


160 x100

= 0.215m aft

 125 
 - 1.5
+

TA = 0.215 x  2  = +0.11m


 125 
 

 125 
 + 1.5
-
-

TF = 0.215 x  2  = - 0.105m


 125 
 

Aft Fwd

© Omar bin Yaakob, July 2006 77


Naval Architecture 2 Notes

T 7.8m 7.8m
(Original)
T 0.11 -0.105
T (Final) 7.91m 7.695m

6.6 Finding LCG of a Ship by measuring draughts

If we have the hydrostatic particulars of the vessel and we can measure the
draughts of the floating ship, we can know the ships weight and LCG.

The steps are as follows:

 Read TF, TA
 Calculate trim = | TF -TA |
 Calculate Tmean = TF + TA
2
 Use Tmean to obtain , MCTC and LCB from hydrostatic table.
 Use trim formula to calculate the value of p i.e.

MCTC x100 x Trim


p =

since p= Distance (LCB ~ LCG), the location of LCG can be determined.

Note that the value of p is the actual distance between B and G in the
longitudinal direction. The actual position of G will depend on the direction
of trim as discussed earlier, see Figure 6.4.

6.7 Lightship survey or Inclining Experiment

The process to obtain the actual mass, KG and LCG of a ship is called
lightship survey and inclining experiment is the part in which the ship is
inclined to obtain KG.
It consists of taking a set of measurements and conducting analysis to
obtain the required objectives. Although sometimes the whole process is
called an inclining experiment, inclining the ship is only a part of the whole
procedure and will achieve only one aspect of the whole objectives. Inclining
experiment itself is very important and required to be done on every ship
under the Merchant Shipping Ordinance 1952.

The main objectives of the lightship survey are to determine for the ship in
as inclined condition and lightship condition:

i. Displacement
ii. KG
iii. LCG

© Omar bin Yaakob, July 2006 78


Naval Architecture 2 Notes

When the three particulars are known for a lightship condition, displacement
and centre of gravity for other conditions can be determined by using loading
calculation as explained in Section 6.5.

Preparation

The measurements are carried out when the ship is completed or nearly
complete. The vessel should be floating freely, not touching the bottom.
Gangways and ladders should be removed. Any loose cables and equipment
must be secured while tanks should either be fully pressed or emptied to
reduce free surface effects.

The draughts are measured at six locations around the ship. Mean draught
is calculated and is used to enter the hydrostatic tables to obtain the
hydrostatic particulars of , MCTC, KMT and LCB. The density of water is
also measured.

The inclining experiment itself is carried out by moving weights across the
ship. The weights are chosen such that the total weight on one side will give
about two degrees of heel. The angles of heel are usually measured using
three pendulums. If other devices are used, one pendulum must still be
used. To increase accuracy, the pendulum should be the longest possible
and to facilitate pendulum deflection reading, the pendulum bob may be
immersed in oil.

Movement of weights

Weights are moved one by one across the deck and after each move
pendulum reading is taken. When all three weights have been moved across,
readings are again taken each time the weights are returned to the original
position.

A B C

D E d F

Figure 6.5

Processing Results

The results are first processed for the ship in the condition at which the
measurements are taken. This is known as the ‘ as inclined condition’.
This condition is different from the final lightship conditions and therefore
corrections will have to be made later.

© Omar bin Yaakob, July 2006 79


Naval Architecture 2 Notes

a. To obtain  and other hydrostatic particulars in as inclined condition

 From six draughts, obtain Tmean.


 Use T mean to read KMT, LCB, MCTC,  and LCF from hydrostatic
tables.
 Hydrostatic tables are prepared normally using the assumed density of
1.025 tonnes/m3. Correction must be made to  and MCTC for density
difference.
For example:
 actual =  table ρ actual
ρ table

b. To obtain KG as inclined

Based on the formula:

wd = GM tan 

By plotting wd vs tan , a straight line is expected as shown in Figure


6.6.

0.04
wd/ 

0.03
0.02
0.01
0
-0.4 -0.2 -0.01 0 0.2 0.4

-0.02 tan 

-0.03
-0.04

Figure 6.6

Since KM is known from hydrostatic data, the height of centre of gravity


above keel,

KG= KM - GM

c. To obtain LCG as inclined


© Omar bin Yaakob, July 2006 80
Naval Architecture 2 Notes

The position of LCG is obtained using methods described in Section 6.6


i.e. by using the trim formula to calculate the value of p i.e.

MCTC x100 x Trim


p==

since p= Distance (LCB ~ LCG), the location of LCG can be determined.

d. Obtaining final , KG and LCG for lightship

A detailed record must be made of items that have yet to be installed


on the ship as well as items that are not part of the ship but present
on the ship during the lightship survey. Examples of the former are
deck equipment, cranes or any other equipment yet to be installed.
Items to be excluded include personnel, inclining weights and other
equipment.

The following format may be used:

Item Mass Kg Vertica Lcg Fwd Aft


(tonnes) (m) l (m from Moment Moment
Momen amidshi (tonne-m) (tonne-m)
t ps)
(tonne-
m)
Ship as
inclined
Items to
remove
Incl.weights -12 12
Personnel -0.7 12.
5
Items to add
Deck +5 17
cranes
Generator +2 1.3
LIGHTSHIP

Example 6.4

Hydrostatic Particulars of MV Penyu LBP 50m is given below:


Drauf Displacement Cb MCTC LCB LCF
tonnes (tonne-m) (m from O) (m dari O)
4.00 5000 0.75 100.00 -2.00 3.0
5.00 6000 0.76 110.00 -1.5 2.0

© Omar bin Yaakob, July 2006 81


Naval Architecture 2 Notes

6.00 7000 0.77 120.00 0.0 0.0


7.00 8000 0.78 130.00 0.0 0.0

The ship M.V. Penyu is floating at level keel draught of 4.50m.

Cargo are loaded as follows:

500 tonnes at lcg 10m Aft of amidships


500 tonnes at lcg 10m Fwd of amidships
500 tonnes at lcg 20m Fwd of amidships
500 tonnes at lcg 15m Aft of amidships

Find its final draughts at the perpendiculars.

SOLUTION:

i. Find displacement (5500)and LCB (-1.75) of original ship. Since


ship is level keel, trim = 0, p= 0 therefore LCG= LCB.
ii. Construct table to find final displacement and LCG.

Item Weight Lcg Moment about


(tonnes) (m from amidship (tonne-
amidships) m)
Ori. 5500 -1.75
ship
Cargo 1
Cargo 2
Cargo 3
Cargo 4
TOTAL

LCG= Total moment amidship = 0.95 m aft of amidship


Total weight

iii. Use total weight to go into Hydrostatic Table and get


Tmean, MCTC, LCB, LCF
6.50m, 125tm, 0, 0

iv. find distance p where p is longitudinal distance between LCB


and LCG.
p = 0.95m

v. Use formula

© Omar bin Yaakob, July 2006 82


Naval Architecture 2 Notes

trim= displacement x p
MCTC x 100

Trim= 0.57m
vi. Use normal formula to calculate delta Ta and delta Tf. Since LCF is
at amidships, delta Ta= delta Tf = trim/2 =

vii. Make final table:

Aft Fwd
Tmean
Delta T
Final 6.785 6.215

Exercise 6

1. A ship LBP 60m has lightship 500 tonnes, KG 3.7m and LCG 2.0m aft of
amidships. The following are loaded:

Item Mass Kg (m) LCG from


(tonnes)  (m)
Fuel Tank 50 0.7 1.5A
Fresh Water 10 3.6 26.2F
Cargo 735 3.2 1.5F
Crew and Store 5 5.5 20.0A

Find Final KG and LCG

2. A ship LBP 100 m has LCF 3 m aft of amidships and floats at 3.2 m and
4.4 m at FP and AP respectively. Its TPC is 10 tonne while MCTC 100 tonne-
m.

The following cargo is added and removed:


UNLOAD 50 tonne cargo from 20 m forward of amidships
30 tonne cargo from 15 m aft of amidships.
10 tonne cargo at amidships.

LOAD 20 tonne cargo at 10m fwd of amidships


5 tonne fuel at 10m aft of amidships

Find the final draughts at the perpendiculars.


Item Weight Lcg (from D (m from LCF) Moment about
amidships) LCF (tonne-m)
Cargo1
Cargo2
Cargo3

© Omar bin Yaakob, July 2006 83


Naval Architecture 2 Notes

Cargo4
Fuel
TOTAL

Use TPC, net weight reduced Rise =


Net moment = tm aft

Trim =

Find deltaTa and deltaTf

TA = m
TF = m

Make final table:

TA TF
Original 4.4
Rise
T
FINAL

2. A ship is being loaded in port. At one point, its draughts are 10.5m A and
12.2m F, MCTC 200tonne-m and LCF 2m fwd of amidships. A further 5000
tonnes cargo is to be loaded at locations 10m fwd of amidships and 10m aft
of amidships. Determine how the cargo should be distributed to complete
loading with an even keel.

3. A ship arrives in port trimmed 25cms by stern. The centre of floatation is


amidships, MCTC 100 tm. A total of 1000 tonnes is to be discharged from
No1 hold (lcg 50m fwd of LCF) and No 4 Hold (lcg 45m aft of LCF). Find how
much to be discharged from each hold for the ship to complete loading on
even keel.

{Solution guide: Find trimming moment currently trimming ship using


Trim (cm) = trimming mmt/MCTC
Remove weights such that net moment from removal will counteract the
trimming moment.}

4. A ship is floating at draughts of 6.1m F and 6.7m A. The following cargo


is then loaded:

20 tonnes with lcg 30m forward of amidships.


45 tonnes 25m forward of amidships
60 tonnes 15m aft of amidships
30 tonnes 3m aft of amidships

© Omar bin Yaakob, July 2006 84


Naval Architecture 2 Notes

If LCF is at amidships, MCTC 200 tonne-m and TPC 35 tonne, find final
draughts.

5. An oil tanker 150m long, displacement 12,500 tonnes, LCF 1m aft of


amidships, MCTC 200 tonnes-m leaves port with draughts 7.2m F and 7.4m
A. There are 550 tonnes of fuel oil in the forward deep tank (centre of gravity
70m forward of LCF) and 600 tonnes in the after deep tank (centre of gravity
60m aft of LCF). During the sea passage, 450 tonnes of oil is consumed from
the aft tank. Find how much oil must be transferred from the forward to the
aft tank if the ship is to arrive on an even keel.

6. A ship arrives in port trimmed 0.3 m by the stern and is to discharge


4,600 tonnes of cargo from 4 holds. 1,800 tonnes of cargo is to be
discharged from No.2 and 800 tonnes from No.3 hold. Centre of floatation is
amidships. MCTC=250 tonne-m.

Centre of gravities of the holds No 1, 2, 3 and 4 from amidships are


45m forward, 25m forward, 20m aft and 50m aft respectively.

Find the amount of cargo to be discharged from Nos. 1 and 4 holds if the
ship is to sail on an even keel.

7. MV Bulker LBP 100 m is floating at a level keel draught of 7m. Its LCG is
4m fwd of amidships.
The following cargo are loaded:
2000 tonne lcg 10m aft of amidships
2000 tonne lcg 10m fwd of amidships 1000 tonne lcg
5m aft of amidships
Find the final draughts at the perpendiculars. Use the provided hydrostatic
curves.

8. MV bulker is floating at draughts of 7.8m F and 7.1m A. Find its


displacement and LCG.

9. Bunga Kintan is floating at draught of 6.2mF and 5.8mA. Its GM was


measured and found to be 0.15m. Find its displacement , LCG and KG.

10. Some weights (w=200 tonnes,lcg= 2m fwd of amidships, kg=5.0m) are


taken off. What is its final displacement , LCG and KG ?

© Omar bin Yaakob, July 2006 85


Naval Architecture 2 Notes

11. Pemerhatian berikut diperolehi daripada ujikaji sendeng. Kirakan sesaran


serta kedudukan pusat graviti membujur (LCG) bagi keadaan kapal kosong.

Drauf: FP 4.92m
Peminggang 5.50m
AP 6.08m

Ketumpatan air laut 1025 kg/m3. Jumlah beban (termasuk pemberat dan air
balas) yang perlu dikeluarkan bagi mendapatkan kapal kosong ialah 354 tonne
pada lokasi 4.0m di depan peminggang dan 10.5m di atas lunas. Panjang kapal
ialah 125m dan butiran hidrostatiknya di dalam air berketumpatan 1025
Kg/m3 adalah seperti berikut;

Drauf Sesaran KMT MCTC LCB LCF


(m) (tonne) (m) (tonne-m) (m dari  (m dari 
) )
6.00 10300 8.4 141.0 3.8m Fwd 0.0 m
5.00 8200 9.0 131.0 4.0m Fwd 0.0 m

© Omar bin Yaakob, July 2006 86


Naval Architecture 2 Notes

HOMEWORK

1. A ship LBP 60m has lightship 500 tonnes, KG 3.7m and LCG 2.0m aft of
amidships.
The following are loaded:

Item Mass Kg (m) LCG FSM


(tonnes) from  (tonne-
(m) m)
Fuel Tank 50 0.7 1.5A 40
Fresh Water 10 3.6 26.2F 15
Cargo 735 3.2 1.5F
Crew and Store 5 5.5 20.0A

Extracts from Hydrostatic Table as follows:

Drauf Sesaran MCTC LCB dari LCF from KMT (m)


(m) (tonnes) (tonne-m)  (m)  (m)
3.6 1280 14.5 0.30A 0.0 4.29
3.8 1320 14.7 0.20A 0.0 4.15

1. Find the final Displacement, KG, LCG and GMT


2. Find Trim
3. Find new Draughts at AP and FP.
4. A 50 tonne weight is moved 6 m across. Find the resulting angle of heel.

© Omar bin Yaakob, July 2006 87


Naval Architecture 2 Notes

Chapter 7 Large Angle Stability

7.1 Righting Lever and Moment

In Chapter 4, we have seen the importance of righting moment to ship stability.


For a ship being acted upon by external moments for example, the righting moment
will balance the external moment giving a steady angle of heel. If there is
inadequate righting moment, the vessel will capsize.

We have also seen that righting moment =  x GZ

Where GZ is the righting lever or righting arm, the perpendicular connecting the
lines of action of buoyancy to the line of action of weight.

External
Heeling W
moment
W
M

G Z
W1 L1

B L
B1
W
K

M

G  Z

  B

Figure 7.1 The righting Lever


GZ

© Omar bin Yaakob, July 2006 88


Naval Architecture 2 Notes

In any loading condition, displacement  does not change. Therefore the righting
moment in any loading condition will only depend on the righting arm GZ.

If we can determine the GZ values in that particular loading condition, we can know
its righting moment and hence the stability of the vessel.

As can be seen from Figure 7.1, GZ is the perpendicular distance between the two
lines of actions; weight through centre of gravity and buoyancy through centre of
buoyancy. Earlier, we have seen that at small angles, the metacentre is considered
stationary. From Figure 7.1, for small angles,
GZ = GMT sin 
where  is the angle of heel.

In any loading condition, GMT is constant. Therefore the righting lever GZ is


directly proportional to sin .

PLOT GZ VS ANGLE OF HEEL

7.2 GZ at Large Angles

The location of the metacentre is no longer stationary at large angles. This is due to
the different shapes of waterplane areas at successive angle of heels. This
differences give rise to different transverse second moments of area. The distance
from the centre of buoyancy to the metacentre is given as

IT
BMT 

At small angle of heel, IT can be assumed constant and hence the metacentre can be
considered stationary. However at larger angles of heels, the waterplane shapes
changes significantly leading to movement of the metacentre. Because of this
reason, the accuracy of the expression GZ=GMT sin  diminishes at large angles. In
other words there is no simple expression relating GZ to GMT .

M’T
M

G 

B’’
B’’
B  B’ 

Figure 7.2 Changing metacentre at large angles

© Omar bin Yaakob, July 2006 89


Naval Architecture 2 Notes

In general, the actual relationship between GZ and  is shown in Figure 7.3. At


small angles, GZ can be considered as directly proportional to Sin . The slope
starts to increase until a point of inflexion is reached. This point marks the angle of
deck-edge immersion. The slope decreases until the maximum is reached. Then
the slope becomes negative and GZ reduces until the ship loses stability at the
angle of vanishing stability.

GZ curve is also called righting arm or statical stability curves. The curve is very
important in stability assessment. Load Line Rule requires GZ curves to be
calculated and the overall stability of the vessel assessed based on the curves.

Point of inflexion
G
Z
(
m Angle of Vanishing Stability
)
GZ value at 20

2 Angle of heel 
0
Range of Stability

Figure 7.3 GZ Curve

7.3 Obtaining Curves of Statical Stability

Curves of statical stability show the variation of GZ at various angles of heels. At


the design stage, the naval architect must ensure that the curves are calculated.
The GZ data is produced at different displacements. Also since KG is required in
the calculation, an assumed KG is used. Later, when the actual  and KG at any
loading condition are known, actual values of GZ can be extracted from the data.

Various methods are used to calculate statical stability curves.

For a wall-sided vessel, GZ can be calculated using a simplified formula.

GZ = ( GM + ½ BM tan 2  ) sin 

Wall-sided formula is only valid for ships which have straight parallel sides. The
formula is valid as long as the deck is not immersed or the bilge is not raised out of
water.

For other types of (real) hullforms, other methods must be used. Details can be
found in Principles of Naval Architecture (Vol 1).

© Omar bin Yaakob, July 2006 90


Naval Architecture 2 Notes

7.4 Cross Curves of Stability

The GZ values calculated at the design stage are presented either in tables or in
Cross Curves of Stability. The GZ values calculated are carried out at only a few
displacements and heel angles. Tables and curves are therefore presented at those
displacements and angles. To obtain values at other displacements or angles, linear
interpolation is used. A typical set of curves are shown in Fig. 7.4.

The value of KG of a ship is not fixed. It changes throughout the ship’s life
depending on how the ship is loaded. The values of GZ will depend on the KG.
However it is difficult to calculate GZ at many displacements, various angles of heel
and different values of KG.

For that purpose, a fixed value of KG is normally used in the design stage. This
assumed position of G is designated S, the height above keel being KS. Since S is
only an assumed centre of gravity, the corresponding point on the line of action of
buoyancy is called N i.e. not Z. The value of the righting lever is thus SN instead of
GZ.

40
SN (m)
50

30
KS= 4.2m
80

15

5

Displacement (tonnes)

Figure 7.4 Cross Curves of Stability

In other words, the cross curves of stability will show plots of the assumed righting
arm, SN rather than the actual righting arm, GZ. To obtain the actual GZ, SN
values at various angles are read at the right displacement, and then corrected for
the correct KG based on Figure 7.5.

© Omar bin Yaakob, July 2006 91


Naval Architecture 2 Notes

M

If S is above G ; GZ = SN + SG Sin 

If S is below G; GZ = SN - SG Sin 

S N

G  Z


B
 K
Figure 7.5

A better method is to put S at the keel i.e. KS=0. In this case SN = KN. The
assumed righting arm, KN values are plotted instead of SN at various displacement.
When G is known for any condition, GZ can be calculated:

GZ = KN – KG Sin 

It must be noted that the values of KG used is the virtual or fluid KG i.e. taking into
consideration the FSC.

7.5 Initial Slope of GZ Curve

At small angles, the expression GZ = GMT Sin  is valid. Also, at small angle, GZ 
GMT  i.e GZ approximates to a straight line with gradient GMT, as shown in
Figure 7.6.
In any loading condition, KMT is constant. Therefore the initial slope will solely
depend on KG.

G
Z

GM

57.3
sudut sendeng 

© Omar bin Yaakob, July 2006 92


Naval Architecture 2 Notes

Figure 7.6 GM as initial slope

© Omar bin Yaakob, July 2006 93


Naval Architecture 2 Notes

Example 7.1

A ship with lightship displacement 1,700 tonnes, KG 3.5m is loaded with 1,800
tonnes of cargo at Kg 3.8m. KMT after loading is 3.8m while KN values are as
follows.

Angle of heel ()


Displacemen 10 20 30 45 60 75
t
(tonnes)
3,000 0.75 1.50 2.16 2.84 3.19 3.26
4,000 0.77 1.54 2.20 2.92 3.25 3.26

Plot the GZ curve and find the area under the curve up to 300.

Solution:

i) Carry out loading calculation to obtain final displacement and KG. (3,500
tonnes, 3.65m)
ii) Find KN values at that displacement.
iii) Correct KN to obtain GZ using GZ = KN – KG sin 
iv) Plot the curve (note that initial slope= GMT=0.15m)
v) Use Simpson rule to find area under the curve up to 30 .
vi) Does the ship pass IMO stability criteria?

(ii) (iii) (v)


KN(m) GZ(m) SM fA
0 0 0 0 0 1 0
10 0.76 0.174 0.634 0.126 3 0.379
20 1.52 0.342 1.248 0.272 3 0.815
30 2.18 0.500 1.825 0.355 1 0.355
45 2.88 0.707 2.581 0.299 sum 1.549
60 3.22 0.866 3.161 0.059
75 3.26 0.966 3.526 -0.266

Area under the curve up to 30 ,

Area = 3 x 10 x  x 1.549 = 0.1014 mrad


8 180

© Omar bin Yaakob, July 2006 94


Naval Architecture 2 Notes

7.6 STABILITY ASSESSMENT

Initial Ship Condition New deadweight


Δ, KG, LCG loading/unloading
w, Kg, lcg

Loading Calculation Table

FINAL Δ,
Use Δ to read
KG, FSC LCG

Calculate Draughts TA
Hydrostatics data and TF
Tmean, LCB, LCF,
Hydrostatics MCTC,
Data KMT Calculate GMT

SN or KN Curves
Curves of Statical
KN/SN Values Adjust for actual KG
Stability
at correct Δ

GZ CURVE
SHIP HULL FORM

Stability
Assessment

IMO Load Line


Criteria
Wind Heeling
Heeling angle
due to heeling
moment

GZ CURVES OF SHIP
WITH NEGATIVE GM DYNAMIC STABILITY

© Omar bin Yaakob, July 2006 95


Naval Architecture 2 Notes

7.6.1 Load Line Criteria.

Small Craft -
Large Ship Fishing Vessel
Stability Criteria Passenger/Cargo
(IMO) (IMO)
(HSC Code)
1. Area Under Curve 0o-  0.07 m.rad if max. GZ
N.A N.A
15o occur at 15 to 30 deg.
2. Area Under Curve 0o-  0.055  0.055 m.rad if max. GZ
 0.055 m.rad
30o m.rad occur at  30 deg.
3 Area Under Curve 0o-40o  0.090 m
N.A  0.089 m.rad
or up to f (flooding Angle) rad
 0.055 + 0.001 (30 -
4. Area Under Curve 15o-
N.A max) if max. GZ occur N.A
30o
between 15 to 30 deg.
5. Area Under Curve 30o-
40o or up to f (flooding  0.03 m rad  0.03 m.rad  0.03 m.rad
Angle)

6. Maximum GZ  0.20 m  0.2 m  0.2 m

7. Angle at Maximum GZ  30.0 deg  15 deg  30 deg

8. Initial GM  0.15 m  0.15 m  0.35 m

Table 1: Stability Criteria for Three Types of Vessel (courtesy


Yahya Samian)

The ship is assessed at a number of loading conditions, for example:

i. Lightship
ii. Homogenous Full Load Departure
iii. Homogenous Full Load Arrival
iv. Ballast Departure
v. Ballast Arrival
vi. Etc.

© Omar bin Yaakob, July 2006 96


Naval Architecture 2 Notes

Example 7.2

A ship lightship 5200 tonnes, KG 7.4m, LCG 2.0m aft of amidships. The
homogenous loaded Departure condition are as follows:

Item Mass VCG LCG from FSM


(t) (m) amidship(m (tm)
)
Fuel Tank 1 1350 1.5 1.2 A 270
Fuel Tank 2 210 3.2 8.0 A 350
Fresh Water 170 7.2 60.0 F 210
Crew and Store 50 9.5 14.0 F -
Cargo No.1 1700 10.1 44.0 F -
Cargo No.2 1900 7.1 17.0 F -
Cargo No.3 1800 8.0 22.0 A -
Cargo No.4 1330 9.0 49.0 F -

Extracts of hydrostatics data at two relevant draughts are as follows:

T Displacement MCTC LCB LCF KMT (m)


(m) (tonnes) (tm ) (m from (m from
) )
8.50 12,994 170.0 2.80 6.80 7.95
9.00 13,999 178.56 3.00 7.088 8.083

KN values at two displacements are as follows:

o 5 10 15 30 45 60 75

13500 0.705 1.412 2.122 4.052 5.555 6.575 7.067


13900 0.707 1.414 2.124 4.054 5.559 6.577 7.071
Find the draughts at the perpendiculars, calculate GMT and check if the vessel pass
the IMO merchant ship stability criteria.

© Omar bin Yaakob, July 2006 97


Naval Architecture 2 Notes

Solution:

Create the final loading table :

Item Mass VCG Mv LCG MA MF FSM


(t) (m) (tm) from (tm) (tm) (tm)
(m)

LIGHTSHIP 5200 7.4 38480 2.0 A 10400 - -

Fuel Tank 1 1350 1.5 2025 1.2 A 1620 - 270


Fuel Tank 2 210 3.2 675 8.0 A 1680 - 350

Fresh Water 170 7.2 1224 60.0 F 10200 - 210

Crew and 50 9.5 475 14.0 F 10200 -


Store

Cargo No.1 1700 10.1 17170 44.0 F - 74800 -


Cargo No.2 1900 7.1 13490 17.0 F - 32300 -
Cargo No.3 1800 8.0 14400 22.0 A 39600 - -
Cargo No.4 1330 9.0 11970 49.0 F 65170

DEADWEIGHT 8510 61426 118270 107800 830

FINAL 13710 7.287 99906 1.522 A 128670 107800 830


DISPLACEMEN
T

F.S.C 830 = 0.0605m


13710

Enter hydrostatic Table at  = 13710 tonnes, to obtain:

TLCF = 8.50 +
13710  12994 (9.00 – 8.50)
13999  12994
716
= 8.50 + x 0.5 = 8.856m
1005

and,

© Omar bin Yaakob, July 2006 98


Naval Architecture 2 Notes

MCTC = 176.1 t.m. ; KMT = 8.045 m;


LCB = 2.949 m.A.
LCF = 7.005 m.A.

© Omar bin Yaakob, July 2006 99


Naval Architecture 2 Notes

For transverse stability

KM = 8.045 m
KG = 7.287

GMSOLID = 0.758 m
F.S.C = 0.061

GMFluid = 0.697m

For longitudinal stability

( p) x 
TRIM =
MCTCx100

= (2.949 - 1.522) X 13710


176.2 x 100

= 1.111 m ke hadapan
. TA = 8.856
-0.501
TA = 1.111 x (72 – 7.005) 8.355 m
144

= - 0.501 m; TF = 8.956
+0.610
9.466 m
1.111 x (72  7.005)
 TF 
144
= + 0.610 m;

o 5 10 15 30 45 60 75

KN (m) 0.706 1.413 2.123 4.053 5.557 6.576 7.069

KGf sin  0.640 1.276 1.902 3.674 5.196 6.364 7.098

GZ (m) 0.066 0.137 0.221 0.379 0.361 0.212 -0.029

KGf = 7.287 + 0.061

GZ = KN - KGf Sin 

© Omar bin Yaakob, July 2006 100


Naval Architecture 2 Notes

Plot GZ curve and calculate and check criteria.

Criteria Actual Required Evaluation


GZ max 0.393m @ 37  0.2m @  30  OK
A30 0.115 m.rad  0.055 m-rad OK
A40 0.187 m.rad  0.09 m-rad OK
A40 – A30 0.072 m.rad  0.03 m-rad OK
GMT 0.697m >0.15m OK

i.e. fulfill all Load Line Rule Criteria

7.6.2 Determining Steady Angle of Heel due to Heeling Moment

When a ship is acted upon by an external or internal moments, the steady angle of
heel can be obtained as the point of equilibrium between the external moment and
the righting moment curve.

From Section 7.1,

Righting moment = Righting Lever x 


=  GZ

Curves of displacement multiplied by GZ values at various angles of heel are called


the righting moment curve, as shown in Figure 7.7.

For example, if a weight w already on board is shifted horizontally from port to


starboard a distance d, the heeling moment is wd Cos θ. The point of equilibrium
where the heeling moment equals the righting moment is the position at which the
ship will heel at a steady angle.

This analysis will also applicable to crane operation. However, in addition to


heeling moment crane operation is also associated with increase in KG i.e.
reduction in GM. The KN correction factor becomes more while the slope becomes
less. Hence the shape of the heeling moment curve reduces , leading to worse
situation.
M E 
o G
m Z
A wdcos
e 
C F
n D
t

E D
O c
Steady Angle of Heel Heel angle 

Figure 7.7 Curve of Righting and Heeling Moments

© Omar bin Yaakob, July 2006 101


Naval Architecture 2 Notes

US Navy Criteria:

For crowding of personnel and lifting of weight:


 Angle at C for lifting of weight or crowding of personnel < 15 degrees
 Heeling mmt at C < 0.6 maximum
 Area A1 > 0.4 total area under curve.

7.6.3 Wind Heeling Moment

Wind heeling moment = k AV2 ℓ Cos2 

Where A = Projected area facing wind


V = Wind speed in knots
l = distance from centroid of wind pressure to centre of
buoyancy.
k = Constant

V
knot
s

By plotting wind heeling moment and righting moment curves, the steady angle of
heel can be obtained.

Moment
(tonne-m) E
 x GZ
k AV2 1 cos2 
A
C D

O c D

Figure 7.8 Steady angle of heel due to constant wind moment

© Omar bin Yaakob, July 2006 102


Naval Architecture 2 Notes

e.g US Navy, stability in wind satisfactory if:


 Heeling moment at intersection of heeling mmt and rignting moment curves <
0.6 maximum.
 Area A1 > 1.4 A2 where A2 extends 25 degress from C to direction of wind.

7.6.4 Determining Safety in Dynamic Mode.

The area under the righting moment curve is the dynamic stability of the vessel.
Dynamic stability defines the amount of energy that the vessel can absorb up to the
heeling angle in question.

When an inclining moment is applied, the baseline of the curve changes as shown
as broken line in Figure 7.7 and Figure 7.8.

With the change in baseline, the stability characteristics of the vessel also changes.
C is the new point of equilibrium (compared to 0º when there is no heeling moment)
where the steady heeling angle is obtained. The ship will return to this angle
whenever additional temporary moment is applied to port or starboard. The new
range of stability to starboard is from C to D. D is the new angle of vanishing
stability, which is less than the original.

Considering Figure 7.7, the area bounded by OAC is the work done by the the
heeling moment to heel the vessel to the steady angle of heel, C. In other words,
this is the potential energy available to heel the vessel from upright to C. If the
moment is applied quasistatically, the vessel will slowly heel to C. However, if it is
applied suddenly, the potential energy will be released suddenly causing the vessel
to overshoot C and reach E. The area OAC equals CEF, and the vessel will
oscillate about C. It is important to ensure that E does not exceed D, the new
angle of vanishing stability.

In the case of wind heeling, similar situation will happen if the wind moment is
applied suddenly as in a gust. Again, from Figure 7.8, the ship will capsize if the
potential energy (area OAC) could not be absorbed fully by the area CDE bounded
by the wind heeling moment curve and the righting moment curves, i.e. E exceeds
D.

7.6.5 Stability of ships with negative GM

A ship with negative GM will not stay upright. It will heel to one side until the
waterplane area is such that metacentric height exceeds the centre of gravity. It will
then stay at this new point of equilibrium, called the angle of loll. In other words a
ship with negative GM will not necessarily capsize.

If the negative GM ship is slowly released from upright (  = 0), it will slowly
approach the angle of loll. However, if the ship is released suddenly, which is
normally the case when GM becomes negative suddenly, the vessel will quickly heel
over by-passing the angle of loll to another angle called the angle of lurch.

© Omar bin Yaakob, July 2006 103


Naval Architecture 2 Notes

Area B

G
Z Angle of vanishing
Angle of Loll stability

Heel Angle 
Area A Angle of Lurch

Figure 7.9 Angle of loll and lurch

The position of angle of lurch will depend on the energy available between upright (
 = 0)and angle of loll (Area A) and the ability of the ship to absorb the energy
beyond the angle of loll (Area B). If the ability to absorb is low i.e. low righting
moment values, the angle of lurch will be big and if it exceeds angle of vanishing
stability, the ship will capsize.

© Omar bin Yaakob, July 2006 104


Naval Architecture 2 Notes

EXERCISE 7 LARGE ANGLE STABILITY

1. For one loading condition, a ship LBP 70m has displacement 1500 tonnes, KG
4.0 m and KM 4.5 m. SN values (with KS = 4.2m) are given in the table below.

Angle () 10 15 20 30 45 60 75
SN (m) 0.23 0.35 0.44 0.44 0.35 0.19 0.01

a. Plot the GZ curve of the vessel.


b. Find area under the curve up to 30 degrees.
c. Check if the ship passes Load Line Rule stability criteria.
d. Check stability condition if a 50 tonne weight already onboard is moved 10
m across.
i.e. check steady angle of heel, area A1 and righting arm at steady angle of
heel.

2. A fishing boat began its trip with displacement of 340 tonnes, KG 3.5 m and KM
3.75 m. After one day journey, the following events occurred :

20 tonne fish loaded (kg 3.0m)


6 tonne fuel consumed (kg 1.0m)
4 tonne fresh water consumed (kg 2.0m)

KM was found to be 3.8 m. The KN values are as follows:

Angle of Heel
Displaceme 10 20 30 45 60 90
nt
(tonne)
300 0.72 1.525 2.40 3.07 3.37 3.55
0
400 0.71 1.521 2.28 2.99 3.29 3.45
0

a. Plot its GZ and righting moment curve


b. In this condition the boat then used the power block to pull the fishing net.
The total weight of net and fish was 10 tonne and the boom of the power
block was 4 m from the boat’s centreline on the starboard side and 5 m above
keel. Plot roughly (but do not calculate) the new righting moment curve and
estimate its steady angle of heel.
c. When at that steady angle of heel, a gust suddenly hit the ship from port. If
the total wind moment is 110 tonne-m, comment on the safety of the vessel.

3. a) Using wall sided formula, show that a wall-sided vessel will loll at an
angle of tan -1 √(2GM/BM).

b) Show the effect of separately increasing beam, increasing KG and increasing


freeboard
on the statical stability curve.

4. With the help of diagrams, explain:


i. What is GZ and why is it important in stability calculations?
ii. Prove that for small angle of heel, GZ = GM sin  .
iii. Why is the above formulae not suitable for large heel angles?
iv. What are SN, KN and GZ curves? Write down relationships between them.

© Omar bin Yaakob, July 2006 105


Naval Architecture 2 Notes

v. Sketch a GZ curve and indicate range of stability, angle of vanishing stability,


initial GM and maximum GZ.

© Omar bin Yaakob, July 2006 106


Naval Architecture 2 Notes

5. A ship 150m has lightship displacement 6500 tonnes, KG 8.0 m, LCG 1.5
m aft of amidships. The ship is loaded as follows :

Item Mass VCG LCG from


(t) (m) amidships
(m)
Fuel Tank 1 1000 1.5 1.2 A
Fuel Tank 2 300 3.2 8.0 A
Fresh water Tank 200 7.2 60.0 F
Crew and Store 50 9.5 14.0 F
Cargo Hold No.1 1800 10.1 44.0 F
Cargo Hold No.2 2000 7.1 17.0 F
Cargo Hold No.3 1600 8.0 22.0 A

* free surface effect is neglected


Extract of hydrostatics data at relevant draughts as follows:

T Displacement MCTC LCB LCF KMT (m)


(m) (tonnes) (tm ) (m dari ) (m dari )
9.50 13,400 176.6 2.85F 1.80A 7.95
10.00 13,500 180.5 3.01F 2.30A 8.08

KN values in meters at two displacements are as follows:

o 5 10 15 30 45 60 75
 (tonnes)
13,400 0.700 1.400 2.023 4.000 5.268 6.346 6.873
13,500 0.706 1.413 2.123 4.053 5.557 6.576 7.069

a. Find Draughts at the perpendiculars


b. Find GMT
c. Check whether the vessel passes stability criteria.
d. Plot the righting arm (GZ) curve of the ship. Estimate its angle of heel when
the ship is lifting a 200 tonne cargo on the port side using its own derrick.
The derrick head is 15 m from the centreline and 15 m above keel.
e.
f. While lifting the cargo, the cable snapped. Find the maximum angle of roll on
the starboard side.

© Omar bin Yaakob, July 2006 107


Naval Architecture 2 Notes

Chapter 8 Flooding and Damage


Stability
8.1 Introduction

Damage stability can be defined as the minimum adequate stability of a ship when
some part of the ship is damaged or opened to the sea.

All types of ships and boats are subject to the risk of sinking if they lose their
watertight integrity whether by collision, grounding or internal accident such as an
explosion. The most effective protection is provided by internal subdivision by
means of watertight transverse and/or longitudinal bulkheads and by some
horizontal subdivision – double bottom in commercial ships and watertight flats in
naval vessels.

A compartment, which has been opened to the sea, is said to have been bilged. It is
necessary to isolate the flooded volume in order to

 restrict loss of transverse buoyancy or build up of such upsetting moment


that capsizing takes place
 restrict change of trim i.e. lost of longitudinal stability
 restrict loss of reserve buoyancy
 restrict the damage to cargo

It is essential to have a standard of subdivision such that there is a reasonable


chance that the ship will remain afloat under such an emergency.

The requirements for subdivision have been fixed by legislature based on the
recommendations of various International Conferences on Safety of Life at Sea.
There are no Government requirements for the number of transverse bulkheads to
be fitted in a cargo ship. However, classification society rules specify the number of
bulkheads required and this is governed by the length of the ship.

Passenger ships – those that carry more than 12 passengers – must comply with
certain standards of subdivision. The method adopted is to determine a line beyond
which the ship should not sink and then ascertain the position and length of the
compartment which when flooded will cause sinkage to that line.This line beyond
which the ship should not sink is known as the Margin Line.

In introduction these concepts, it is necessary to establish certain basic definitions.


The most important and relevant of these are

Bulkhead Deck

The bulkhead deck is the uppermost deck to which the transverse watertight
bulkheads extend (usually the main deck).

Margin Line

The margin line is a line drawn parallel to, and three inches (76 mm) below, the
bulkhead deck at the side.

© Omar bin Yaakob and Mohamad Pauzi Abdul Ghani, July 2006
108
Naval Architecture 2 Notes

Permeability

The percentage volume of a space that can be flooded is known as the permeability.

available volume

total volume

When a compartment is flooded, it is rare for the total volume of this compartment
to becompletely filled with water. This is because the compartment will already
contain certain equipment or stores depending upon its use.

The table below from “Basic Ship Theory - 4th Edition” by Rawson & Tupper lists
some typical ship compartment permeabilities.

Permeability (%)

Watertight Compartment ( Warship) 97


Watertight Compartment ( Merchant
95
ship)
Accommodation Spaces 95

Machinery Compartments 85

Stores or Cargo Holds 60

Dry Cargo Spaces 70

Floodable Length

The maximum length of a compartment, which can be flooded so as to bring a


damaged ship to float at a waterline tangential to the margin line.

Curve of Floodable Length

This is a curve, which, at every point in its length, has an ordinate representing the
length of the ship, which may be flooded with the centre of the length at that point
and without the margin line being submerged.

Bonjean Curves

The area of the transverse section of a ship to successive waterlines can be


calculated and plotted as a curve showing the variation of sectional area with
draught. The curves are frequently drawn on the ships profile at the displacement
stations or on a centre line with those for stations in the fore body on the right hand
side and for the after body on the left hand side. They enable the displacement and
LCB to be calculated for any waterline, trimmed or even keel.

© Omar bin Yaakob and Mohamad Pauzi Abdul Ghani, July 2006
109
Naval Architecture 2 Notes

8. 2 The Effects of Damaged Compartments

A damaged ship could be lost in one of several ways.

 If the ship is left with inadequate maximum righting moment or dynamical


stability, it could simply be overwhelmed by the seaway and the weather.

 If the angle of list or trim is too great, placing non-watertight parts of the ship
underwater, then additional flooding will occur. In this case the ship could
lose
transverse stability, roll over and capsize

 Longitudinal stability could also be lost in a similar manner causing the ship
to plunge (go down bow or stern first). One of the most notable examples of
plunging is the Titanic.

 A ship may be lost even if stability is not compromised. It may simply sink.
This is called foundering.

Table 1 describes the damage scenarios and their effect to the ship

TABLE 1 : Damage Scenarios and Their Effect to the Ship

Location of
Case Effect to the Ship
Damage

Long : At Centre of 1. Parallel singkage – Increase Draft


Floatation, LCF 2. Increase or reduce GM depending on
I
Trans : Symmetry about Waterplane Area.
Centre Line 3. Reduce Freeboard parallel to the keel.

1. Parallel Singkage – Increase Draft


Long : Frd or Aft of LCF
2. Trim by the Bow or Aft of Ship
II Trans : Symmetry about
3. Reduce Freeboard significantly at bow or
Centre Line
aft.

Long : At Centre of 1. Parallel Singkage – Increase Draft


Floatation, LCF 2. Heeled to one side
III
Trans : Port or Starboard 3. Reduce GM and GZ
Side 4. Reduce Freeboard Significantly at one side

© Omar bin Yaakob and Mohamad Pauzi Abdul Ghani, July 2006
110
Naval Architecture 2 Notes

1. Parallel Singkage – Increase Draft


Long : Frd or Aft of LCF 2. Trim by the Bow or Aft of Ship
Trans : Port or Starboard 3. Heeled to one side
IV
Side 4. Reduce GM and GZ
(Worst Scenario) 5. Reduce Freeboard Significantly at bow or
aft and worst at one side.

8.3 Methods of Calculations

This section discusses the fundamental behavior of a damaged ship and introduces
two techniques that allow its analysis

 The Added Weight Method


 The Lost Buoyancy Method.

3.1 Flooding Calculation Methods

Firstly, define the degree of flooding to be examined (often given by regulations).

Then find
 the waterline, trim and heel for the damaged floating condition (is there
sufficient reserve buoyancy left to prevent foudering?)
 the damaged stability ( is there sufficient stability left to prevent capsizing
and plunging?)

Two techniques for flooding calculation explained on a rectangular vessel with a


damaged central compartment.

8.3.1 The Added Mass Method

Wo A B Lo

W L

E  F

WL Intact waterline
WoLo Damaged waterline
ABFE Added mass due to flooding
WoWLLo Additional buoyancy required

In order to calculate the added mass it is necessary to guess the damaged draught
AE and verify (trial and error)

© Omar bin Yaakob and Mohamad Pauzi Abdul Ghani, July 2006
111
Naval Architecture 2 Notes

8.3.2 The Lost Buoyancy Method

Wo A B Lo

W L
C D

E  F

WL Intact waterline
WoLo Damaged waterline
CDFE Lost of buoyancy i.e. must be made up by the buoyancies of WoACW
and BLoLD

The lost of CDFE can be calculated exactly. The additional buoyancy up to WoLo
can be found from parallel immersion/sinkage of the damaged waterplane
(excluding the portion AB).

8.3.3 Determining Draughts after Damage

The effects of bilging a mid-length compartment may be shown most simply by


considering a box barge of length L, breadth B and draught d having a mid-length
compartment of length l, permeability .

L
B L

l
If this compartment is bilged, buoyancy is lost and must be replaced by
increasing the draught. The volume of buoyancy lost is the volume of the
compartment up to waterline WL, less the volume of water excluded by the
cargo in the compartment.

Volume of lost buoyancy =  l BT

© Omar bin Yaakob and Mohamad Pauzi Abdul Ghani, July 2006
112
Naval Architecture 2 Notes

This is replaced by the increase in draught multiplied by the area of the intact
part of the waterplane, i.e. the area of waterplane on each side of the bilged
compartment plus the area of cargo which projects through the waterplane in
the bilged compartment.

Area of intact waterplane = (L- l l B(1-)


= LB- l l B- l B
= (L- l)B

Increase in draught = volume of lost buoyancy


Area of intact waterplane

=  l BT
(L- l )B

=  lT
L- l

 l may be regarded as the effective length of the bilged compartment.

Example 1

A box barge 30m long and 8 m beam floats at a level keel draught of 3m and
has a mid-length compartment 6m long. Calculate the new draught if this
compartment is bilged:
a) with  =100%
b) with  = 75%

(a) Volume of lost buoyancy = 6 x 8 x 3m3


Area of intact waterplane = (30-6) x 8m2

Increase in draught = 6 x 8 x 3 m3
24 x 8

= 0.75m
New draught = 3.75m

(b) Volume of lost buoyancy = 0.75 x 6 x 8 x 3m3


Area of intact waterplane = (30-0.75 x 6) x 8 m3

Increase in draught = 0.75 x 6 x 8 x 3


25.5 x 8

= 0.529m
New draught
= 3 + 0.529
= 3.529 m

© Omar bin Yaakob and Mohamad Pauzi Abdul Ghani, July 2006
113
Naval Architecture 2 Notes

8. 4 STABILITY AFTER FLOODING OF AMIDSHIPS COMPARTMENT

Stability is initially assessed by calculating metacentric height, GM T.


Example 8.2

A box-shaped vessel LBP = 30m, B =10 m floats at level keel draught of 2 m in


saltwater. Its KG = 3m. An amidship compartment, length 15m is damaged and
flooded. Calculate its final draught and transverse GM.

ORIGINALLY

 = L x B x T x density = 30 x 10 x 2 x 1.025 = 615 tonnes

GM = KM-KG =(KB + BMT) - KG= (1.0 + 4.17) -3.0 = 2.17m

COMPARTMENT NOW BILGED (FLOODED)

Lost Buoyancy = 15 x 10 x 2 = 300 m3

Complete Waterplane Area = 300 m2

Lost Waterplane Area = 150 m2

Intact Waterplane area = 150m2

Parallel Sinkage i.e. T = lost buoyancy = 300 = 2.00 m


Intact waterplane 150

Hence New Draught = 4.00 m

Two methods are normally used for calculation of metacentric heights during
flooding calculations. The methods uses two different basis but finally yield same
answers on the effect of flooding.

a. USING ADDED MASS BASIS

This method deals with the problem as if the hull is intact and an amount of water
is poured into the hull.

 = 30 x 10 x 4 x 1.025 = 1230 tonne

1
KB =  4.00 = 2.00 m
2

100
BM = = 2.083 m
12  4.00

The mass of water added (added mass) = 1230 – 615 = 615 tonne

615(3  2)
KG = = 0.500 m
1230
© Omar bin Yaakob and Mohamad Pauzi Abdul Ghani, July 2006
114
Naval Architecture 2 Notes

15  103
FSC = = 1.042 m
30  10  4  12

Final KGf = 3.542 m

GMf = 4.083 – 3.542 = 0.541 m

b. USING LOST BUOYANCY BASIS

This method considers the damaged and flooded compartment to be part of the sea
and has nothing to do with the intact vessel.

T
KB = = 2.00 m
2
BM = (30-15) x 103 = 2.083 m (actual volume of displacement unchanged)
12 x 15 x 10 x 4
KM = 4.083 m

KG = 3.000 m (unchanged since water not part of content of


the ship)

GM = 1.083 m (note a different GM obtained)

Apparently, there are two different GM . However, actual measure of stability is


righting moments.

For example, check righting moment at 3 heel.

a) Added Mass method,  =1230 tonnes and GM =0.541m


Righting Moment =  x GM sin  = 1230 x 0.541 x sin 
= 34.83 tonne-m
b) Lost Buoyancy method,  = 615 tonnes and GM =1.083m
However the righting moment =  x GM sin 
= 615 x 1.083 x sin 3 = 34.83 tonne-m

i.e. similar righting moment, both methods give similar measure of stability

Exercise:

A box-shaped vessel LBP = 120m, B =20 m floats at level keel draught of 8 m in


saltwater. Its KG = 7 m. An amidship compartment (μ = 0.8) , length 25m is
damaged and flooded. Calculate its final draught and transverse GM using lost
buoyancy and added mass methods.

Show that both methods give the same assessment of the vessel’s initial stability.

© Omar bin Yaakob and Mohamad Pauzi Abdul Ghani, July 2006
115
Naval Architecture 2 Notes

8.5 FLOODING OF END COMPARTMENTS AND CHANGE OF DRAUGHTS

Flooding of end compartments will induce parallel sinkage as well as trim,


leading to large changes to draughts at the perpendiculars.

Example 3

A box shaped vessel 140m long, 20m wide floats in sea water at a draught of 4.5m.
A compartment extending the full width of the vessel with bulkheads at locations
15m and 40m forward of AP is flooded. The area and volume permeability for the
compartment are 75% and KG is 8.75m.

Calculate draughts and GM.

140m
140
20m m

4.5
m
40m
15m

Lost buoyancy  25  20  4.5  0.75  1687.5 m 3


Intact WPA  140  20 - 25  20  0.75  2425m 2
1687.5
Parallel sinkage i.e. T   0.696m
2425

a  q
Shift in LCF, p   6.57 m fwd
Intact WPA
where a  lost WPA
q  distance of lost WPA to original centroid

lost buoyancy  Distance LCFnew to lost WPA centroid   L


Trim 
I Fnew

© Omar bin Yaakob and Mohamad Pauzi Abdul Ghani, July 2006
116
Naval Architecture 2 Notes

Original second moment of area about old LCF,


B  L3
I Fold   4,573,333 m 4
12

New second moment of area about new LCF,



I Fnew  I Fold  WPA  p 2   i  a p  q 
2

where i  2 nd moment of area of lost WPA and a is its area
I Fnew  m4

1687.5  49.07  140


Trim   m by stern

76.57 63.43
TA    1.36 m TF  Tp    m
140 140

Tf (m) Ta (m)
Original 4.5 4.5
Parallel 0.696 0.696
sinkage
T
FINAL

To calculate GM,

Lost buoyancy Basis;

5.196
KB   2.598 m
2
1/12140 - 25  20 3
BM   6.08 m
140  20  4.5
GM  KB  BM - KG  8.678 - 8.75  - 0.072 m

Example 5

A ship LBP 150m, breadth 36m was involved in a collision and one of its
compartment ( = 0.95) length 36m with its centre 37m fwd of amidships is
damaged and open to sea. Just before collision, draughts of 3.1m and 7.3m were
recorded at forward and aft respectively. With these draughts the following data
were obtained from table of hydrostatics particulars.

 = 22166 m3 BML = 320 m


AW = 4820 m2 LCF = 2.5 m Aft of amidship
© Omar bin Yaakob and Mohamad Pauzi Abdul Ghani, July 2006
117
Naval Architecture 2 Notes

CB = 0.8

38
At centre of hold, original draught = 3.1  (7.3  3.1)  4.2 m
150
 Lost buoyancy = 36 x 36 x 0.95 x 0.8 x 4.2 = 4137 m3

Lost wpa = 36 x 36 x 0.95 = 1231 m2

Intact wpa = 4820 – 1231 = 3589 m2

4137
Parallel sinkage =  1.2 m
3589

Shift in LCF = lost wpA x dist to orig. F


Intact wpa

1231  (37  2.5)


p =  13.55 m aftwards
3589

Trimming moment = lost buoyancy x dist. to new F


= 4137 x (13.55 + 2.5 + 37)
= 219468 m4

Original second moment of are about LCF,


IFold = BML x  = 320 x 22166 = 7,093,120 m4

IFnew = {IFold + WPA x p2}- {i +  a (p + q)2 }

where i = 2nd moment of area of lost waterplane and a is its


area.

IFnew = 4, 380, 145 m4

219468
Trim = Trimming moment x L = 150  7.52 m by head
4380145
IFnew

91
TF = 7.52   4.56 m
150

59
TA = 7.52   2.96 m
150

Tf (m) Ta (m)
Original 3.1 7.3
Parallel 1.2 1.2
sinkage
T 4.56 -2.96
FINAL 8.86 5.54

© Omar bin Yaakob and Mohamad Pauzi Abdul Ghani, July 2006
118
Naval Architecture 2 Notes

© Omar bin Yaakob and Mohamad Pauzi Abdul Ghani, July 2006
119
Naval Architecture 2 Notes

8.6 Mass and Centroid of Water entering ship bringing it to a particular


Waterline

Example 6

A ship of length 158 m has a displacement of 18,500 tonnes and the centre of
buoyancy 2.8 m abaft amidships. At a waterline tangential to the margin line the
areas of the immersed sections are as follows:

Stn AP 1/2 1 11/2 2 3 4 5 6 7 8 81/2 9 91/2 FP


Area 35 80 115 150 179 213 218 215 210 190 133 94 56 23 0

Determine the mass of water that has entered the ship and the distanceof its
centroid from amidships.

Solution

Stn Area SM f(A) Lever fm(A)


AP 35 ½ 18 5 90
½ 80 2 160 41/2 720
1 115 1 115 4 460
11/2 150 2 300 31/2 1050
2 179 11/2 269 3 807
3 213 4 852 2 1704
4 218 2 436 1 436
5 215 4 860 0 5267 Aft
6 210 2 420 1 420
7 190 4 760 2 1520
8 133 11/2 200 3 600
81/2 94 2 188 31/2 658
9 56 1 56 4 224
91/2 23 2 46 41/2 207
FP 0 ½ 0 5 0
4680 3629 Fwd
Difference 1638 Aft

1
Displacement   15.8  4680  1.025  25,200 tonnes
3
1638
LCB   15.8  5.53 m aft amidships
4680

Displacement Lcb Moment


(tones)
Final Condition 25,200 5.53 A 139,350
Initial Condition 18,500 2.80 A 51,800
Difference 6,700 87,500

© Omar bin Yaakob and Mohamad Pauzi Abdul Ghani, July 2006
120
Naval Architecture 2 Notes

87550
Centroid   13.07 m aft amidships
6700
 mass of water that has entered the ship  6,700 tonnes
centroid of water  13.07 m aft amidships

8.7 Location of Damaged Compartment bringing it to a particular


Waterline

Example 7

A ship of length 128 m and displacement 12,200 tonnes with the centre of
buoyancy 0.77 m forward of amidships is brought, as the result of damage,to a
waterline at which the displacement is 14,200 tonnes and the centre of buoyancy is
7 m forward of amidships. The damage opens to the sea a compartment bounded by
transverse bulkheads and with a permeability of 80%. The areas of the immersed
sections for the fore body, in the damaged condition, at equidistant stations
commencing at amidships, are 139,148,158, 162, 139, 84 and 0 m2.

Determine the length and position of the damaged compartment.

Solution

Displacement (tonnes) LCB (m) Moment


Initial condition 12,200 0.77 F 9390
Damaged condition 14,200 7.00 F 99400
Water in compartment 2,000 45m F 90010
2000  0.975
Volume of compartment   2435 m 3
0.8
Assume a mean immersed sectional area of 120 m 2
2435
 length of compartment   20.3 m
120

Assume bulkheads say 9 m aft and 11.3 m forward of centroid of flooded


compartment.

© Omar bin Yaakob and Mohamad Pauzi Abdul Ghani, July 2006
121
Naval Architecture 2 Notes

Immersed sectional area (m )

100
2

AP
45 m fwd amidships

Immersed area, As SM f(As) Lever fm(As)


Aft End 158 1 158 0 0
124 4 496 1 496
Fwd End 65 1 65 2 130
719 626

Volume of compartment  1/3  719  10.15  2432 m 3


626
Centroid of compartment   10.15  8.9 m fwd of aft end of compartment;
719
this more or less satisfies the requiremen ts
 Length of compartment  20.3 m
Aft end of compartment  45 - 9  36 m fwd of amidships
Fwd end of compartment  36  20.3  56.3 m fwd of amidships

8.8 Direct Flooding Calculations and Floodable Length

Δo
Δ1
L1
W1
G g
Lo
Wo
B1
Bo

b x w

© Omar bin Yaakob and Mohamad Pauzi Abdul Ghani, July 2006
122
Naval Architecture 2 Notes

Figure shows the profile of a ship with waterline W1L1 tangential to the margin line.
The purpose of the calculation is to determine the extent and position of the
flooding which will bring the ship from waterline WoLo to the waterline W1L1.

Let

WoLo waterline of undamaged ship Δo and Bo the corresponding


displacement and
centre of buoyancy

W1L1 waterline tangential to the margin line with Δ1 and B1 the


corresponding displacement and centre of buoyancy

g centroid of lost buoyancy w

G centre of gravity of ship for both conditions

Then

Mass of water gaining access to ship or the lost of buoyancy, w = Δ1 – Δo

Taking moments about Bo


Δ1 . b = w . x

1  b
x
w

Consequently the extent of the lost buoyancy or of the added weight to the4
waterline W1L1 and the position of the centroid can be determined. Thus, the
volume of water admitted is 0.975δΔ and the total volume of the compartment is
given by
0.975w
v

The length of the compartment is derived from a curve of areas of immersed


sections as follows:

l1

Portion of curve of immersed area

Figure shows a portion of such a curve derived from the Bonjean curves at the
waterline W1L1 which is tangential to the margin line. The centroid of the added
weight or of the lost buoyancy is on the ordinate at A. It is then necessary to

© Omar bin Yaakob and Mohamad Pauzi Abdul Ghani, July 2006
123
Naval Architecture 2 Notes

determine an area under the curve, which will have its centroid on this ordinate at
A and also represent the volume

0.975w
v

The process is one of trial and error.

The procedure is to estimate a mean ordinate generally less than the ordinate at A,
say A1 and the first approximation to the length l1 of the compartment is given by

v 0.975w  100
l1  
A1   A1
where  is expressed as a percentage.

This length should be laid off so that the middle is on one side or the order of the
ordinate at A according to the shape of the curve. The volume and position of the
centroid corresponding to the length l1 can be determined by Simpson’s rule; this is
used as the basis for a second approximation. Normally the correct length and
position are obtained at the second attempt.

The length of the compartment so determined is known as the Floodable Length as


it is the length in the region considered which may be flooded without making the
ship sink beyond the margin line.

By calculations as indicated above for a series of waterlines which are tangential to


the margin line at different points throughtout the length of the ship it is possible to
determine a series of values for the plotting of a set of curves of floodable length as
shown below.

G
B

E F C A D
Aft Engine Fwd
space room space

Length of ship

Floodable Length Curve

In the above Figure, the point A is the mid-length of a compartment of length


represented by the ordinate AB. The vertical and horizontal scales used for plotting
the curves of floodable length are the same. Thus, CA = AD = 1/2AB and the
tangent of the angles BCA and BDA is 2.

© Omar bin Yaakob and Mohamad Pauzi Abdul Ghani, July 2006
124
Naval Architecture 2 Notes

It is thus possible to ascertain whether any chosen length of compartment at any


position exceed the floodable length, by plotting the isosceles triangle with the
length of the compartment as the base.

Thus the length of the compartment represented by EF exceeds the floodable length
since the apex G lies above the curve.

Exercise 8

1. A box barge 100m long, depth 10m and 20 m beam floats at a level
keel draught of 8m and has a mid-length compartment 20m long. If
KG=8.4m, comment on stability of this vessel if the compartment is
bilged?

2. A box shaped vessel 150m long, 14m wide floats in sea water at a
draught of 7.0m. A compartment extending the full width of the vessel
with bulkheads at locations 35m and 65m forward of amidship is
flooded. The area and volume permeability for the compartment are
75% and KB can be assumed as equal to KG.

Determine whether the bow of the vessel will be submerged and


calculate GM using lost buoyancy basis.

3. A ship of length 150 m has a displacement of 20,000 tonnes and the


centre of buoyancy 1.0 m aft of amidships.

At one waterline tangential to the margin line the areas of the


immersed sections are as follows:

Stn AP 1 2 3 4 5 6 7 8 9 FP

Area 0 60 135 200 220 220 215 205 175 110 30

Find its floodable length and its centroid, assuming a permeability of


80%.

© Omar bin Yaakob and Mohamad Pauzi Abdul Ghani, July 2006
125
Naval Architecture 2 Notes

© Omar bin Yaakob and Mohamad Pauzi Abdul Ghani, July 2006
126

Potrebbero piacerti anche